Re: [obm-l] Curvas

2003-07-22 Por tôpico Alexandre Tessarollo

Ambas as curvas são hipociclóides. Como disse o Dirichlet, são obtidas através do 
rastro de um ponto fixo numa circunferência pequena rodando dentro de uma maior. Só 
para não deixar margens a dúvidas, vale ressaltar que são circunferências tangetes 
internamente.

A figura 1 é um hipociclóide tricúspide ou, se vc preferir, um deltóide. Possui 
diversas propriedades interessantes - por exemplo, a envoltória das tangentes é um 
outro deltóide. Tb está intimamente ligado ao desenho da elipse.

Já a figura 2 especificamente eu não conheço. Se tivesse que chutar diria que não 
possui nenhum nome exclusivo. Contudo, certamente tem algum nome genérico e 
propriedades genéricas.

Se vc realmente estiver interesado, posso depois vasculhar meus alfarrábios para te 
dar maiores detalhes de como construir, propriedades, equações, curvas relacionadas, 
etc. Só pedir que eu venço minha preguiça :)

[]'s

Alexandre Tessarollo
-- 
__
Sign-up for your own FREE Personalized E-mail at Mail.com
http://www.mail.com/?sr=signup

CareerBuilder.com has over 400,000 jobs. Be smarter about your job search
http://corp.mail.com/careers

=
Instruções para entrar na lista, sair da lista e usar a lista em
http://www.mat.puc-rio.br/~nicolau/olimp/obm-l.html
=


Re: [obm-l] Como os Matemáticos Complicam II

2003-07-11 Por tôpico Alexandre Tessarollo
Meu caro João Paulo,

Pelo que entendi você não quer exatamente a resolução de um problema quando o coloca 
aqui - vc quer sim saber de onde diabos aquilo surgiu e por que razão vc deveria 
resolvê-lo.

Aviso logo que este é um e-mail longo. Desde já minhas desculpas a todos e em 
particualr ao N pelo off-topic (nem tão off assim) - espero que esse meu e-mail ajude 
a esclarecer as coisas e a encerrar essa discussão.

**

Matemática,pra mim não tem a ver com o lado emocional.Só não consigo
entender bem uma coisa que não estou vendo um motivo lógico pra
existir(Caso de raiz,x e y etc)

Vc vê motivo lógico para os poemas de Vinícius de Moraes existirem? Assim como os 
poemas, raízes são desdobramentos e evoluções de determinados campos do conhecimentos, 
de determinadas formas de expressão. Tanto a matemática como a poesia são formas do 
ser humano se expressar e são campos de conhecimento, em constante evolução. Às vezes 
é difícil entender como surgiu um Vinícius de Moraes, mas se considerar toda a base 
literária e musical que veio antes, um Vinícius torna-se natural.

Assim com os poetas, assim com a matemática. Talvez o que esteja lhe faltando seja uma 
boa aula de história da matemática. Aí vc vai entender o que veio de one. Desde o 
pastor contando suas ovelhas no pasto até os nomes esquisitos que vc vê pro aqui, como 
integrais e equações diferenciais.

Ah sim, só para deixar um teaser: E qual o motivo lógico para existir Astronomia? De 
que me serve saber que a nebulosa de sei-lá-o-que é composta disso e daquilo outro? E 
para que Música? Qual a utilidade de se tocar um instrumento ou ficar criando novos 
instrumentos?

**

É muito mais fácil eu entender um texto todo em japonês.Pelo menos
posso ir atrás de um dicionário e traduzir.
Já com matemática,não tem aonde ir.

Tem sim. Livros de matemática, professores, colegas que saibam um pouco mais que você 
e fóruns como esse. Contudo, é sempre mais fácil se você usar abordagens mais sutis. 
Assim como você não pede uma informação na rua dizendo mil palavrões e gritando a 
altos brados que aquele bairro não deveria existir, tb com matemáticos é mais 
apropriado um tanto de gentileza. Afinal, somos todos humanos.

Eu mesmo já tive vários alunos que me disseram que não gostavam de matemática. Nunca 
tive problemas com isso. É como dizer ao fã de uma banda que vc odeia aquele grupo. Se 
os dois conseguirem se expressar claramente e fundamentar seus pontos de vista, vcs 
certamente não vão brigar. Podem não mudar de opinião, mas vcs vão entender pq o outro 
gosta/não gosta tanto assim da banda.

Outra coisa que ajuda muito é aprender o básico da língua da pessoa. Se vc chega para 
um londrino e pergunta onde á a pizzaria mais próxima ele não vai entender lhufas. 
Já se vc disser Pizza! Where? Where pizza? ele, com um tanto de boa vontade, vai 
entender e te apontar algo como one two three right then one two left para dizer que 
é a terceira quadra à direita e então a segunda à esquerda. Sei que o exemplo parece 
bobo, mas acho eu que ilustra bem.

**

Já expûs como descobrir de onde vieram os problemas e conceitos matemáticos (curso de 
história da matemática) e o porquê da dificuldade de conseguir ajuda em matemática 
(abordagens mais gentis e predisposição favorável sempre ajudam a se conseguir o que 
se quer de um humano). Falta dizer agora por que diabos vc deveria resolver probs de 
matemática.

Bem, afora os motivos mais óbvios e terrenos - passar na escola, no vest, etc - 
existem alguns outros:

1) Desenvolver e treinar o raciocínio lógico

Resolver problemas de matemática é como tentar pintar um desenho respeitando as 
bordas. Achar que x=4 ou ter um desenho pintado não serve de muita coisa, mas saber 
observar um problema, saber dizer quais são os dados, saber escolher a melhor 
abordagem, conseguir implemntar essa abordagem e achar a resposta são coisas que 
treinam o grande músculo que é nosso cérebro. Ficamos com um raciocínio lógico 
melhor - que nos permitirá depois calcular que não vai dar para chegar no outro bairro 
em menos de dez minutos *e* conseguir entradas para o cinema *portanto* vamos naquela 
outra sala mais perto. Mesma coisa para a aula de artes - ninguém tem um diploma de 
que conclui o curso de respeitar as bordas com louvor, mas é algo que treina sua 
coordenação motora fina e sua capacidade de concentração e estimula sua acuidade 
visual.

2) Resolver problemas práticos

Se vc vai preparar um receita e na hora vê que só tem metade de determinado 
ingrediente, vc precisa saber que só poderá usar a metade de todos os outros 
ingredientes para manter a proporcionalidade.

Já se vc pega um receita que dá para 8 pessoas e quer preparar para 20, vc precisa ter 
noção de que usará duas vezes e meia todos os ingredientes.

Mesmo um professor de português precisa saber calcular suas médias - e nem sempre é 
tão fácil. Muitas escolas trabalham com um sistema de pesos diferentes para cada nota 
que, quando chega no final do semestre, 

Re: [obm-l] POLEMICA

2003-07-11 Por tôpico Alexandre Tessarollo
Operação 1:
Compra por 20 e vende por 30. Até aqui lucro líquido de 10. O sujeito tira 20 do bolso 
e ganha 10 extras - nesses termos são 50% de lucro sobre o preço de compra.

Operação 2:
Recompra por 50 e revende por 60. Aqui ele entra com todo o anterior (20 iniciais mais 
10 de lucro inicial) mais 20 do próprio bolso e sai com 60, sendo (20 iniciais) + (10 
de lucro inicial) + (outros 20 do bolso) + (10 de lucro novo). Nessa operação o lucro 
líquido é de 10. Se vc consicerar como investimento apenas os 20 que ele tira do 
bolso, o lucro aqui foi de 50%.

O investimento total do cara (o que saiu do bolso dele) foi 40. Ele lucrou (10 
iniciais) + (10 na segunda operação). Portanto, teve lucro de 50% com base no 
investimento total.

Uma outra maneira de pensar o problema seria ver que ele investiu (investir=tirar do 
próprio bolso) 40 e no final recebeu 60 de crédito. Portanto, 60-40=20 de lucro. 
Novamente, 50% sobre o investimento total.


[]'s

Alexandre Tessarollo
-- 
__
Sign-up for your own FREE Personalized E-mail at Mail.com
http://www.mail.com/?sr=signup

CareerBuilder.com has over 400,000 jobs. Be smarter about your job search
http://corp.mail.com/careers

=
Instruções para entrar na lista, sair da lista e usar a lista em
http://www.mat.puc-rio.br/~nicolau/olimp/obm-l.html
=


[obm-l] Re: PAs de ordens1

2002-11-22 Por tôpico Alexandre Tessarollo

   Antes de mais nada, obrigado pelas respostas do N para os raios e de todos que 
responderam às questões do somatório de x^2 e da PA de k-ésima ordem. Gostaria de 
comentar a resposta do Domingos Jr. em particular:

   Ou, mais genericamente, como se calcula a soma do n primeiros termos de uma PA de 
2a ordem, onde b[n+1]-b[n]=a[n], sendo a[n] o termo de uma PA normal(de 1a 
ordem)? Naturalmente temos a[1], R e b[1].

o somatório de 1 até n dos a[i] vai dar: n.a[1] + somatório{i = 1 até n-1}[(n-i).b[i]]

a1 = a1
a2 = a1 + b1
a3 = a1 + b1 + b2
...
an = a1 + b1 + b2 + ... + b[n-1]
a1 + a2 + ... + an = n.a1 + (n-1).b1 + (n-2)b2 + ... + b[n-1]

   Ok, acho q vc assumiu que b[i] era a PA de primeira ordem e a[i] seria a PA de 
segunda ordem. Considerando assim, está perfeitamente corrto o que vc escreveu. Nos 
meus rascunhos eu tinha chegado exatamente até esse ponto. O problema que eu tive foi 
expressar a soma da PA de 2a ordem em função APENAS de a[1], b[1], n e R (razão da PA 
d 1a ordem).

   Concordo que a sua resposta faz isso IMplicitamente, mas eu gostaria de algo 
EXplícito. 

   Por analogia, poderíamos dizer que o somatório da PA de 1a ordem de b[1] até b[n] 
pode ser escrito como S[n]=(b[1]+b[n])n/2, mas eu prefiro expressar explicitamente, ou 
seja, S[n]=(2b[1]+(n-1)R)n/2.

   Desculpem a falta de clareza anterior. Ah sim, ainda falta alguém se manifestar 
quanto a generalização dessa pergunta, ou seja, como expressar o somatório de uma PA 
de k-ésima ordem em função (explícita :-)) dos primeiros termos de cada PA de ordem 
inferior?

[]'s

Alexandre Tessarollo
-- 
__
Sign-up for your own FREE Personalized E-mail at Mail.com
http://www.mail.com/?sr=signup

One click access to the Top Search Engines
http://www.exactsearchbar.com/mailcom

=
Instruções para entrar na lista, sair da lista e usar a lista em
http://www.mat.puc-rio.br/~nicolau/olimp/obm-l.html
O administrador desta lista é [EMAIL PROTECTED]
=



[obm-l] PAs de ordens1

2002-11-21 Por tôpico Alexandre Tessarollo

   Estou num momento de diarréia mental. Qual é e como deduzir a fórmula de somatório 
de x^2, para x=1,2,..,n? 

   Ou, mais genericamente, como se calcula a soma do n primeiros termos de uma PA de 
2a ordem, onde b[n+1]-b[n]=a[n], sendo a[n] o termo de uma PA normal(de 1a ordem)? 
Naturalmente temos a[1], R e b[1].

   Generalizando ainda mais, sejam a{1}[1], a{2}[1],..,a{k}[1] respectivamente os 
primeiros termos de PAs de 1a, 2a,..,k-ésima ordem e R a razão da PA de primeira 
ordem. Em função desses parâmetros, qual a soma dos n primeiros termos da PA de 
k-ésima ordem?

[]'s

Alexandre Tessarollo
-- 
__
Sign-up for your own FREE Personalized E-mail at Mail.com
http://www.mail.com/?sr=signup

One click access to the Top Search Engines
http://www.exactsearchbar.com/mailcom

=
Instruções para entrar na lista, sair da lista e usar a lista em
http://www.mat.puc-rio.br/~nicolau/olimp/obm-l.html
O administrador desta lista é [EMAIL PROTECTED]
=



[obm-l] Re: exponencial

2002-11-04 Por tôpico Alexandre Tessarollo
Resolva:
[( raiz quadrada de 3) + 1]^x + [( raiz quadrada de 3) - 1]^x = 8


Graficamente vejo duas soluções: uma positiva ( x = 2)e outra
negativa.
Me pediram algebricamente. Divido com vocês a dor de cabeça.

=+=+=+=+=+=+=+=+=+=+=+=+=+=+=+=+=+=+=+=+=+=+=+=+=+=+=+=+=+=+


 Nooossa, quanto tempo faz que não escrevo aqui... Bem, de qq forma, vamos lá, de cara 
partindo p/a grosseria: Vou usar o Binômio de Newton, que diz que
(x+a)^n=sum[p=0, n]{Cn,p*(a)^p}*(x)^(n-p))
Leia-se ... igual a somatório, de p=0 até p=n, de ...

 No nosso caso, vou expandir (sqrt3+1)^x + (sqrt3-1)^x de uma só vez. Fica:
sum[p=0, x]{Cx,p*(1)^p*(sqrt3)^(x-p)} + sum[p=0, x]{Cx,p*(-1)^p*(sqrt3)^(x-p)}

Juntando num mesmo somatório e colocando a combinação e o sqrt3 em evidência
sum[p=0, x]{Cx,p*(sqrt3)^(x-p)((1)^p+(-1)^p)}

Devido ao (1)^p+(-1)^p, podemos afirmar que o somatório só vai considerar as parcelas 
com p=2k. Substituindo p por 2k e admitindo x PAR, temos:
sum[k=0, x/2]{Cx,2k*(sqrt3)^(x-2k)*(2)} =
= 2*sum[k=0, x/2]{Cx,2k*(sqrt3)^(x-2k)}

 Se x for ímpar, o somatório vai de k=0 até k=(x-1)/2. Até aqui eu acho que está tudo 
certo. Agora pretendo calcular os somatórios em separados, isto é, calcular o sum de 
Cx,2k e depois o de (sqrt3)^(x-2k).

Primeiro, o de Cx,2k. Montando o triângulo de Pascal, temos

1
1 1
1 2 1
1 3 3 1
1 4 6 4 1
etc

Vale lembrar que o somatório de Cx,p é a soma da linha e que, o número i da linha é 
dado por i=x+1. (Lembra que x era o expoente dos nossos binômios?)
Eliminando os termos da forma Cx,2k+1, temos

1
1 
1 1
1 3 
1 6 1

Creio que seja fácil ver que a soma da linha i é 2^(i-2).Ou, em outras palavras, digo, 
variáveis, o somatório de Cx,2k vale 2^(x-1).
 
Agora, calculemos o de (sqrt3)^(x-2k). Se vc substituir os valores iniciais de k, verá 
que se trata de uma PG. Calculando a soma dessa PG num rascunho, achei (3^(x/2+1)-1)/2.

Tomando o somatório que tínhamos e substituindo pelos valores encontrados
2*sum[k=0, x/2]{Cx,2k*(sqrt3)^(x-2k)}=
= 2*sum[k=0, x/2]{Cx,2k}*sum[k=0, x/2]{(sqrt3)^(x-2k)}=
= 2*2^(x-1)*(3^(x/2+1)-1)/2 =
= 2^(x-1)*(3^(x/2+1)-1)

Bem, lembrando nós só trabalhamos o lado esquerdo da igualdade proposta pelo problema, 
podemos igualar o nosso resultado ao lado direito da igualdade. Teremos:

2^(x-1)*(3^(x/2+1)-1)=8 PARA X PAR (lá no meio dos somatórios eu supus que x era par, 
lembra?)

Para x ímpar, basta fazermos algumas alterações. Se não errei nada, fica
2^(x-1)*sqrt3*(sqrt3^(x+1)-1)=8


 E é no momento em que lembramos que x=2 era raiz que eu lembro porque tinha parado de 
escrever e-mails na madruga :-) Se vc fizer x=2 na equaçào de x par, vc acha que 
16=8!! Ou eu acabei de revolucionar completamente a matemática ou, o que é mais 
provável, cometi algum erro crasso nas minhas contas. Já revisei-as umas 3 vezes, mas 
não consegui detectá-lo. Alguém se habilita?

[]'s

Alexandre Tessarollo
-- 
__
Sign-up for your own FREE Personalized E-mail at Mail.com
http://www.mail.com/?sr=signup

=
Instruções para entrar na lista, sair da lista e usar a lista em
http://www.mat.puc-rio.br/~nicolau/olimp/obm-l.html
O administrador desta lista é [EMAIL PROTECTED]
=



[obm-l] Re: dificuldade

2002-08-18 Por tôpico Alexandre Tessarollo


   Admitindo que a ordem em cada banco importa, isto é, existe o lugar à direita e o à 
esquerda no banco, basta fazermos uma permutação circular com todas as 24 pessoas. A 
fórmula para permutação circular de n é (n-1)!, logo, existem 
23!=25.852.016.738.884.976.640.000 maneiras desse povo subir na roda gigante.

   Já se vc estiver se importando apenas com que dupla vai se sentar aonde, ou seja, 
tanto faz quem vai sentar de um lado ou de outro, temos 2 maneiras: A primeira é 
dividr o resultado anterior por 2^12. (Vale lembrar que uma vez escolhida a dupla, 
existem 2 maneiras desses dois se sentarem AB e BA). Portanto, 
23!/2^12=6.311.527.524.141.840.000 maneiras.

   A segunda maneira seria separar primeiro as duplas 
[C(12,2)*C(10,2)*C(8,2)*C(6,2)*C(4,2)*C(2,2)] e depois fazer a permutação circular de 
12 duplas [11!]. Multiplica um número pelo outro e vc terá o mesmo resultado de antes.

[]'s

Alexandre Tessarollo
-- 
__
Sign-up for your own FREE Personalized E-mail at Mail.com
http://www.mail.com/?sr=signup

=
Instruções para entrar na lista, sair da lista e usar a lista em
http://www.mat.puc-rio.br/~nicolau/olimp/obm-l.html
O administrador desta lista é [EMAIL PROTECTED]
=



[obm-l] Re: mais que CRUEL

2002-08-16 Por tôpico Alexandre Tessarollo
 as dagonais V2V3, V5V8 e V6V7 são as únicas paralelas a V1V4. Some os 
ínidces dos vértices de cada diagonal. Vc obterá 5 ou 13 nesses resultados, certo? 
Detalhe importante, nós estamos num OCTÓgono (8) e 13=5+8, certo?

   A partir daí eu elaborei uma conjectura para polígonos regulares mas ainda não 
consegui enunciá-la formalmente. Algo como: 

Numere, no sentido anti-horário, os vértices de V1 até Vn. Tome um daigonal ViVj 
qualquer, com ji (por praticidade). As diagonais paralelas a ViVj serão da forma 
V[i+k]V[j-k], para (i-j)/2k(j-i)/2 EM Z[n] (ou seja, sempre fazendo a congruência 
módulo n).

   É, algo assim... Aguardo comentários.

   Ah, antes que eu me esqueça, sabemos que S1+S2+S3+S4=total de intersecções entre 
dua diagonais distintas quaisquer. Ou seja, 
S1+S2+S3+S4=[n(n-3)/2]*[n(n-3)/2 -1]/2
S1+S2+S3+S4=n(n-3)[n(n-3)-2]/8

Sabendo S1, S2 e S4, é fácil calcular o S3 que eu tinha pulado antes. O único prob é 
efetivamente calcular o S4...

[]'s

Alexandre Tessarollo
-- 
__
Sign-up for your own FREE Personalized E-mail at Mail.com
http://www.mail.com/?sr=signup

=
Instruções para entrar na lista, sair da lista e usar a lista em
http://www.mat.puc-rio.br/~nicolau/olimp/obm-l.html
O administrador desta lista é [EMAIL PROTECTED]
=



[obm-l] Re: CRUEL

2002-08-15 Por tôpico Alexandre Tessarollo



   Estive fazendo umas contas e creio que posso ter chegado a uma resposta parcial. 
Por uma questão de praticidade, numerei os vértices de A[0] até A[n-1], no sentido 
anti-horário.

   Tome o vértice A[0] e vá ligando com os outros vértices (A[1]; A[2]; ...; A[n-1]). 
Note que cada diagonal A[0]A[i] divide o plano em dois semi-planos. Se o polígono é 
convexo, podemos determinar exatamente quantos vértices ficam em cada semi-plano.

   Para quem está com a figura, é fácil ver que, dada a diagonal A[0]A[i], todos os 
pontos A[j] com 0ji estão em um semi-plano e todos os pontos com ijn estão em 
outro semi-plano. Qualquer diagonal com vértices em semi-planos distintos terá uma 
intersecção com a diagonal A[0]A[i].

   Logo, cada diagonal possui SUM[i=1, n-1]{(i-1)(n-1-i)/2} 
Leia-se somatório de (i-1)(n-1-i)/2 quando i varia de 1 até n-1

   Como são n diagonais, bastaria calcular n*SUM. Contudo, cada intersecção está sendo 
contada duas vezes (uma para cada diagonal que a produz). Assim, o resultado seria 
n*Sum/2.

   Devo confessar que não terminei as contas do somatório pq estou meio cansado, mas 
creio que esse seja o número de intersecções pedido. Aguardo comentários...

PROBLEMA:

(Admitindo que o valor encontrado antes esteja correto :-))

   O número encontrado antes NÃO necessariamente é o número de intersecções DISTINTAS. 
Pode-se afirmar com certeza que o número de distintas é menor ou igual ao valor 
calculado.

   O problema agora é: Como descobrir quais intersecções são múltiplas, isto é, 
formadas por mais de 2 diagonais? E como contá-las?


[]'s

Alexandre Tessarollo
-- 
__
Sign-up for your own FREE Personalized E-mail at Mail.com
http://www.mail.com/?sr=signup

=
Instruções para entrar na lista, sair da lista e usar a lista em
http://www.mat.puc-rio.br/~nicolau/olimp/obm-l.html
O administrador desta lista é [EMAIL PROTECTED]
=



[obm-l] Re: Maio01

2002-05-13 Por tôpico Alexandre Tessarollo

Ok, mil desclupas. As palavras e os números se embolaram na minha frente... Agora 
lendo o enunciado com a devida 
calma, temos, no sentido anti-horário: AB=45; BP=45; PC=20; CD=20; DM=30; MA=30. 

Trace PD. Observe que o triângulo PCD é isósceles. Como o angPCD+angABP=180º, 
temos cos(PCD)=-cos(ABP). 
Projetando C ortogonalmente em AB, temos um triângulo retângulo do qula tiramos 
cos(ABP)=5/13, logo, cos(PCD)=-5/13. 
Aplicando Lei do Cossenos no triângulo PCD, temos PD=120/sqrt(5).

Aplicando Lei dos Senos em PCD, temos CP/sen(PDC)=PD/sen(PCD). Daí tiramos que 
sen(PDC)=sqrt(5)/5. Como 
angPDC+angPDM=90º, temos cos(ODM)=sen(PDC)=sqrt(5)/5. Aplicando Lei dos Cossenos em 
PDM, teremos 
PM=6sqrt(145).

Se eu não errei nenhuma conta no caminho, o resultado é esse. Outro opção seria 
tomaro o triângulo isósceles ABP ao 
invés do PCD. Depois, em vez de observar PDM, trabalha-se com PAM, mas o raciocínio 
todo é basicamente o mesmo.

[]'s

Alexandre Tessarollo

PS: Obrigado ao Antônio pelo sen(18). Já tinha ouvido o valor várias vezes mas nunca 
tinha parado p/pensar a respeito nem 
nunca tinha visto nenhuma demonstração. Valeu mesmo.




Desculpe, mas o problema não fornece figura alguma, eu o passei como me foi 
fornecido. A única coisa que se sabe sobre o 
ponto P é que ele se localiza em BC, tal que BP meça 45, por conseguinte PC 20.

 - Original Message -
 From: Alexandre Tessarollo [EMAIL PROTECTED]
 To: [EMAIL PROTECTED]
 Sent: Saturday, May 11, 2002 5:28 AM
 Subject: [obm-l] Re: Maio01


   2-No trapézio ABCD, o lado DA é perpendicular às bases AB e CD. A base AB mede 45, 
 a base CD mede
   20 e o lado BC mede 65. Seja P no lado BC tal que BP mede 45 e seja M o ponto 
 médio de DA.
  
   Calcule a medida do segmento PM.

-- 
___
Sign-up for your own FREE Personalized E-mail at Mail.com
http://www.mail.com/?sr=signup

=
Instruções para entrar na lista, sair da lista e usar a lista em
http://www.mat.puc-rio.br/~nicolau/olimp/obm-l.html
O administrador desta lista é [EMAIL PROTECTED]
=



[obm-l] Re: Maio01

2002-05-11 Por tôpico Alexandre Tessarollo

 Pessoal vocês poderiam me ajudar nessas duas questões da olimpíada de maio de 2001 ? 
 1-Em volta de um círculo situam-se dez moedas de 1 cm de raio . Cada moeda é 
tangente ao círculo e
 às duas moedas vizinhas. Demonstre que a soma das áreas das dez moedas é o dobro da 
área do
 círculo. 


   Junte os centros das moedas e vc terá um decágono regular de lado 2cm (2r para 
generalizar). Junte os ponto de tangência das moedas com o círculo e vc terá outro 
decágono regular, de lado L e inscrito numa circunferência de raio R. Observe o 
trapézio formado pelos centros (O1 e O2) de duas moedas consecutivas e seus 
respectivos pontos de tangência com o círculo (T1 e T2). Temos o trapézio O1T1T2O2. 
Sabemos que o ângulo interno de um decágono regular é 144º. (Odeio essa frase, mas) é 
fácil ver que T1O1 e T2O2 bissectam os ângulos O1 e O2 do decágono maior. Logo, NO 
TRAPÉZIO, os ângulos O1 e O2 são de 72º. A base maior O1O2 é 2cm (2r).

   Projetando T1T2 em O1O2 e usando cosseno nos dois triângulos retângulos que 
sobram, vemos que T1T2 é igual à 2-2cos(72) [2r-2cos(72), na nossa generalização]. 
Assim nós temos o valor do lado do decágono menor. Pelas propriedades de polígonos 
regulares, sabemos que L/2R=sen(180/n), onde R é o raio da circunscrita e n é o número 
de lados. No decágono menor, L=2-2cos(72) [2r-2cos(72)]. Aplicando essa fórmula e 
fazendo as contas, temos R=(r-sen(18))/sen(18).

   Queremos provar que 2*Pi*R^2=10*Pi*r^2. Ou seja, queremos mostrar que R^2=5*r^2. 
Fazendo as devidas substituições e simplificando, só precisaremos saber o seno de 18º. 
Só que esse vale (sqrt(5)-1)/4. Simplificando devidamente, veremos que, para r=1, a 
afirmação é válida, CQD.

  Talvez eu não tenha sido claro o suficiente em alguns pontos pq estou com sono 
agora, mas os resultados que eu não msotrei são facilmente comprováveis - vide a 
relação eentre o lado do polígono regular e o raio da circunscrita. O único resultado 
que eu usei e não sei mostrar é o valor do sen(18). Esse eu realmente colei  :)))

 2-No trapézio ABCD, o lado DA é perpendicular às bases AB e CD. A base AB mede 45, a 
base CD mede
 20 e o lado BC mede 65. Seja P no lado BC tal que BP mede 45 e seja M o ponto médio 
de DA. 

 Calcule a medida do segmento PM. 


   Tá, DA=65. Mas cadê o ponto P? Meu e-mail atual é meio esquisitão, então nem sei se 
vc mandou um attach. Dei uma olhada no arquivo da lista e lá não tem anexo nenhum... 
Será q vc poderia descrever a posição de P?
 Obrigado 

 Marcus Dimitri 

[]'s

Alexandre Tessarollo
-- 
___
Sign-up for your own FREE Personalized E-mail at Mail.com
http://www.mail.com/?sr=signup

=
Instruções para entrar na lista, sair da lista e usar a lista em
http://www.mat.puc-rio.br/~nicolau/olimp/obm-l.html
O administrador desta lista é [EMAIL PROTECTED]
=



[obm-l] Re: ???

2002-05-01 Por tôpico Alexandre Tessarollo


Bem, ele planta a primeira muda, anda 5m, planta a 2a, anda +5m, planta
a terceira e volta 10m até a 
origem. Logo, sendo d[n] a distância percorrida p/plantar o n-ésimo trio
de árvores, temos d[1]=20m.

P/plantar a 4a muda, ele anda 10+5m, depois +5 p/a 5a, +5 p/a sexta
e volta +25m até a origem. Ou 
seja, d[2]=50m.

Se vc observar, d[n]=2d[n-1]+2*(5+5+5) ou seja:
d[n]=2*(d[n-1]+15).

Assim, temos
d[1]=20
d[2]=50
d[3]=130
d[4]=290
d[5]=610
d[6]=1250
d[7]=2550  ...

Confesso que não consegui achar nenhum padrão nessa sequencia, a não ser
que
d[n]=3d[n-1]-d[n-2]  para n2
d[n]=8d[n-2]-3d[n-3]para n3

Então, tentemos de uma forma mais simples:
os primeiros 10m ele percorre 2n vezes no trio n de mudas (10*2*n);
os 15m seguintes ele percorre 2(n-1) vezes (15*2*(n-1));
os 15m depois, 2(n-2) vezes (15*2*(n-2));
e assim sucessivamente.

Lembrando que há 21 trios de mudas, a distância total percorrida seria
de
D=10*2*21+15*2*20+15*2*19+...15*2*1
D=420+15*2*(20+19+18+...+1)
D=6720

Vale lembra que, após plantar a última muda o jardineiro NÃO precisa
voltar a origem. Portanto, 
devemos excluir a distância entre a última e a primeira muda, ou seja, precisamos
excluir 100m da conta - 
se alguém não entedeu pq 100m, imagine 21 MARCOS numa estrada, distantes
5m uns dos outros e 
tente achar a distância total.

Assim, a distância percorrida pelo nosso jardineiro do plantio da primeira
muda até o plantio da última 
muda é de 6620m ou, se vc preferir, 66,2 hectômetros(a abreviação é hm ou
Hm?)

Se você puder colocar a resoluçào daquelas pessoas que acharm 65 seria
legal. Obviamente, uma das 
2 está errada e a outra talvez esteja certa. De qq forma, comentem a minha
resposta e vamos ver como 
se acha 65...

[]'s

Alexandre Tessarollo

PS: NÃO tenho certeza dessa solução, ainda vou revisá-la com a devida calma.
Achei melhor publicá-la 
logo pq sei q depois de revisá-la num papel não vou ter a necessária paciência
p/digitar... :-)) A 
propósito., minhas resolução e resposta bateram com a sua? Caso não, publique-as
vc tb!

Eder wrote:

   Olá colegas de lista, O seguinte problema,proposto em um vestibular da
UNB,está causando uma 
  controvérsia lá no colégio...Tenho um gabarito dizendo que a resposta
é 65,porém eu não consigo chegar a 
  esse resultado,nem alguns colegas de sala.Se alguém puder resolver,agradeço.

No projeto urbanístico de 
  uma cidade ,o paisagista previu a urbanização do canteiro central de uma
das avenidas,com o plantio de 
  63 mudas de Flamboyant,todas dispostas em linha reta e distantes 5m uma
da outra.No dia do plantio,o 
  caminhão descarregou as mudas no canteiro central,no local onde seria
plantada a primeira muda.Um 
  jardineiro foi designado para executar o serviço.Para isso,partindo do
lugar onde as mudas foram 
  colocadas,ele pegou 3 mudas de cada vez,plantou-as nos locais 
designados,enfileirando-as
uma após a 
  outra.Calcule ,em hectômetros,a distância total mínima percorrida pelo
jardineiro após finalizar o trabalho.



=
Instruções para entrar na lista, sair da lista e usar a lista em
http://www.mat.puc-rio.br/~nicolau/olimp/obm-l.html
O administrador desta lista é [EMAIL PROTECTED]
=



Re: [obm-l] Estudos sobre Equações

2002-04-23 Por tôpico Alexandre Tessarollo



[EMAIL PROTECTED] wrote:

 Olá amigos..
 Ai vão alguns problemas interessantes de equações..
 Se puderem me dar uma luz...

 1-
 O número de raízes reais da equação
 x.(x + 1).(x² + x + 1) = 42



 2-
 O número de raízes reais da equação
 3x^4 - 2x³ + 4x² - 4x + 12 = 0



 Há para essas equações 1 e 2 alguma critério ?

 3-
 A diferença entre a maior e a menor raiz da equação
 (x - 2)(x - 3)(x - 4)(x - 5 ) = 360


Admitindo que exista x inteiro como solução, temos que escrever 360 como
produto de 4 inteiros consecutivos.
1*2*3*4=24
2*3*4*5=120
3*4*5*6=360Logo, uma raiz é x=8.

Vale lembrar que (-3)*(-4)*(-5)*(-6)=360, logo, x=-1 tb é raiz. Se vc
desenvolver o produto e arrumar a eq, vc deverá ter
x^4-14x^3+71x^2-154x-240=0.
Agora que já conhecemos duas raízes, podemos baixar o grau dessa eq até um de
segundo grau, que deverá ser x^2-7x+30=0. Observando essa eq, ela só admite
raízes complexas.

Logo, às raízes de maior e menor valor são, respectivamente, 8 e -1, e
sua diferença é 9. Todavia, deve haver uma forma melhor de garantir que as
outras duas raízes são complexas... Alguém se habilita?





 4-
 A diferença entre a maior e a menor raiz da equação
 (x² + x + 1)(2x² + 2x + 3 ) = 3(1 - x - x²)

Hum... De olho só sei dizer que zero é raiz... No braço, sei dizer que só
existe uma outra raiz real e que ela está entre -2 e -1, se eu não errei as
contas. Mas não sei como tirar essa outra ou como mostrar, de forma
tragável que as outras dua raízes são complexas...



 São todos exercícios muito bons , com conhecimento a nível de 1° grau ,
 eu não consegui enxergar uma solução válida.

Ih! Oops... Só valia 1º grau? Hum... Bem, isso não permite muita coisa...
Contudo, a solução da 3 ainda é válida, posto q eles já sabem Báskara (ou
método de resolução de eqs do 2º grau, p/os puristas ;-)) O problema é q um
aluno de 1o grau só vai ter duas raízes e não vai sequer ter noção de q
existem outras Na grosseria e na inocência, ele vai poder responder a
pergunta da questão...



 Obrigado..
 Rick Barbosa

[]'s

Alexandre Tessarollo

=
Instruções para entrar na lista, sair da lista e usar a lista em
http://www.mat.puc-rio.br/~nicolau/olimp/obm-l.html
O administrador desta lista é [EMAIL PROTECTED]
=



[obm-l] Geometria dos balcãs

2002-04-23 Por tôpico Alexandre Tessarollo


Da outra lista, um pouco de diversão...

[]'s

Alexandre Tessarollo

PS: Ainda não so li com a devida calma, mas acho que falta uma parte do
enunciado do primeiro prob retirado da 3a olimpíada...


==

   Date: Tue, 23 Apr 2002 02:41:41 -0700
   From: Antreas P. Hatzipolakis [EMAIL PROTECTED]
Subject: BMO

The Balkan Mathematical Olympiads:

 1st - 16th: http://bmo.ournet.md/previous.html

   17th: http://bmo.ournet.md/index.html

Geometry Problems (selection):

Let O be the circumcenter of a triangle ABC, D be the midpoint of
the side AB and E be the centroid of the triangle ACD. Prove that
the lines CD and OE are orthogonal if and only if AB=AC.

   The 2nd Balkan Mathematical Olympiad
   1985, Sofia, Bulgaria

---

A line that passes through the incenter I of the triangle ABC meets
the incircle in D and E and the circumcircle of the triangle ABC in
F and G (D is between I and F). Prove that , where r is the radius
of the incircle. When does the equality hold?

Let ABCD be a tetrahedron and points E, F, G, H, K, L be situated
on the edges AB, BC, CA, DA, DB, DC respectively. Prove that if

   AE*BE = BF*CF = CG*AG = DH*AH = DK*BK = DL*CL

then the points E, F, G, H, K, L are placed on a sphere.

   The 3rd Balkan Mathematical Olympiad
   1986, Bucharest, Romania

---

Let A1B1C1 be the orthic triangle of an acute-angled nonequilateral
triangle ABC and A2, B2, C2 be the contacts of the incircle of the
triangle A1B1C1 with its sides. Prove that the triangles A2B2C2 and
ABC have the same Euler line.

  The 7th Balkan Mathematical Olympiad
  1990, Sofia, Bulgaria

---

Three circles Gamma, C1 and C2 are given in the plane. C1 and
C2 tangent Gamma internally at points B and C, respectively.
Moreover C1 and C2 tangent each other externally at a point D.
Let A be one point in which the common tangent of C1 and C2
intersects Gamma. Denote by M the second point of intersection
of the line AB and the circle C1 and by N the second point of
intersection of the line AC and the circle C2. Further denote by
K and L second points of intersections of the line BC with C1
and C2, respectively. Show that lines AD, MK and NL are
concurrent.

   The 14th Balkan Mathematical Olympiad
   Kalampaka, Greece, April 29, 1997

---

Given an acute triangle ABC, let D be the midpoint of the arc BC of
the circumcircle around the triangle ABC, not containing the point A.
The points which are symmetric to D with respect to the line BC and
the circumcentre O are denoted by E and F, respectively. Finally,
let K be the midpoint of the segment EA. Prove that:
  a) The circle, passing through the midpoints of the sides of
the triangle ABC, also passes through K;
  b) The line, passing through K and the midpoint of the segment
BC is perpendicular to the line AF.

  The 16th Balkan Mathematical Olympiad
  Ohrid, FYR Macedonia, May 7th, 1999

---

APH

=
Instruções para entrar na lista, sair da lista e usar a lista em
http://www.mat.puc-rio.br/~nicolau/olimp/obm-l.html
O administrador desta lista é [EMAIL PROTECTED]
=



Re: [obm-l] Equação do 2°

2002-04-20 Por tôpico Alexandre Tessarollo

Bem, sabemos que as raízes são (-b+sqrt(delta))/2ae
(-b-sqrt(delta))/2a. Trabalhando só nos reais, podemos dizer que a primeira é
sempre maior ou igual à segunda. Logo, a diferença das duas é:

(-b+sqrt(delta))/2a-(-b-sqrt(delta))/2a=
=(-b+sqrt(delta)+b+sqrt(delta))/2a=
=2sqrt(delta)/2a=
sqrt(delta)/a


Na equação específica que vc pediu, fica

sqrt[(2+sqrt3)^2-4(7+4sqrt3)(-2)]/(7+4sqrt3) =

=sqrt[4+3+4sqrt3+56+32sqrt3]/(7+4sqrt3)=
=sqrt[63+36sqrt3]/(7+4sqrt3)=
=3sqrt[7+4sqrt3]/(7+4sqrt3)

Esse resultado até é bonitinho, mas se vc quiser racionalizar, fica

3sqrt[7+4sqrt3](7+4sqrt3)

Bem, se eu não errei nenhuma conta, é isso aí... Confiram!

[]'s

Alexandre Tessarollo



(7 + 4 sqrt3)x^2 +(2 + sqrt3)x - 2 = 0

[EMAIL PROTECTED] wrote:

 Olá amigos..

 Caro Aderbal..
 A seguinte questão do quadrado que possui um ponto interior que dista 10
 cm de dois vértices e 10 cm do lado , a resolução que eu lhe mandei ,acho
 que esta correta ?Mais posso ter me enganado em algum lugar ..mais a idéia
 , acho que é mais ou menos como esta lá..
 Vou tentar lhe mandar a figura com tudo direitinho .

 Rafael, obrigado pela dica , vou tentar passar a figura para esse formato.

 E para não perder a viagem , vai ai um exercício de equação do 2°.

 1-A diferença entre a maior e a menor raiz da equação (7 + 4 sqrt3)x^2 +
 (2 + sqrt3)x - 2 = 0

 Obs:
 Eu tentei fazer pela Soma e pelo Produto , tipo:
 Achava a soma o produto , e depois montava um sistema com eles , explicitava
 na soma x1 e substituía no produto , para encontrar x2 , tentei fazer no
 braço , mais não consegui .
 Será que existe alguma relação entre a diferença de raízes ?(Estilo a Soma
 -b/a e produto c/a)
 Grato..
 Rick Barbosa

 --
 Use o melhor sistema de busca da Internet
 Radar UOL - http://www.radaruol.com.br

 =
 Instruções para entrar na lista, sair da lista e usar a lista em
 http://www.mat.puc-rio.br/~nicolau/olimp/obm-l.html
 O administrador desta lista é [EMAIL PROTECTED]
 =

=
Instruções para entrar na lista, sair da lista e usar a lista em
http://www.mat.puc-rio.br/~nicolau/olimp/obm-l.html
O administrador desta lista é [EMAIL PROTECTED]
=



Re: [obm-l] Resolução Geo Plana exerc. 3

2002-04-20 Por tôpico Alexandre Tessarollo



[EMAIL PROTECTED] wrote:

 Aderbal,aí vai novamente a resolução, se algum coléga da lista puder opinar
 ...
 Grato..

 Como AP = PD = PE

 Lado do   Q(ABCD) = 10 + h(APD)

L=10+h
h=L-10

Portanto, teremos um triângulo retângulo de hipotenusa PD=10 e catetos h=L-10 e
L/2. Fazendo o Pitágorás, fica:

10^2=(L-10)^2+(L/2)^2
100=L^2+100-20L+L^2/4
0=5L^2/4-20L
5L(L/4-4)=0

L/4-4=0
L/4=4
L=16


Alguém concorda? Alguém discorda? Alguém não entendeu o que eu fiz?
Manifestem-se...

[]'s

Alexandre Tessarollo

=
Instruções para entrar na lista, sair da lista e usar a lista em
http://www.mat.puc-rio.br/~nicolau/olimp/obm-l.html
O administrador desta lista é [EMAIL PROTECTED]
=



[obm-l] Re:

2002-04-18 Por tôpico Alexandre Tessarollo



Adherbal Rocha Filho wrote:

   Olá pessoal,
 Vocês poderiam me ajudar com ests questões?

 1. P é um pnt interior a um quadrado ABCD.As distancias de P aos
 verices A e D e ao lado BC são iguais a 10.O lado do quadrado mede?

Sugiro uma figura, mas se vc estiver com preguiça, o ASCII deve quebrar
um galho

AB

E   PF

D C

E é o ponto médio de AD. Como AP=DP, podemos afirmar que P está na
mediatriz de AD e, logo, EP é perpendicular a AD. F é a intersecção de EP com
BC. Sabmeos que AP=DP=PF=10.

Pela figura, é fácil ver que o lado do quadrado x=AB=EP+PF=EP+10. Logo,
x=EP+10; EP=10-x. AE=x/2. Por Pitágoras, temos:

AE^2+EP^2=AP^2
(x/2)^2+(10-x)^2=10^2
x^2/4+100+x^2-20x=100
5x^2/4-20x=0
5x(x/4-4)=0

x/4-4=0
x=16





 2. Só mais esta:
 Determine as soluções reais de x^2=2^x
 Obrigado!


Essa eu já vi diversas similares mas até hoje não aprendi a fazer esse
tipo de questão... Mas, se for te ajudar, x=2 é uma soluçào óbvia do
equação. Olhando pelo gráfico de x^2 e 2^x (um tanto similar a da exp(x)),
vemos que eles se cortam em apenas dois pontos. Resta agora achar o outro.

Parêntesis
Momento infame e infeliz daquele professor q não sabe responder: Pronto,
já resolvi metade do prob com x=2 e indiquei o caminho para a segunda raiz.
Agora o resto é com vc
Fim do(s) parêntesis

[]'s

Alexandre Tessarollo

=
Instruções para entrar na lista, sair da lista e usar a lista em
http://www.mat.puc-rio.br/~nicolau/olimp/obm-l.html
O administrador desta lista é [EMAIL PROTECTED]
=



Re: [obm-l] TN

2002-04-11 Por tôpico Alexandre Tessarollo


Responde sim, obrigado. E desculpas a todos pelo reçiproca... :-)

[]'s

Alexandre Tessarollo

Luiz Alberto Duran Salomao wrote:

 Caro Alexandre:
 A reciproca nao eh verdadeira.
 Contra-exemplo: 111 eh composto (pois eh divisivel por 3) mas 3 nao eh
 composto.
 Acho que isto responde tambem aas suas outras questoes, nao eh ?
 Abracos,
 Luiz Alberto Salomao

 Alexandre Tessarollo wrote:

  Luiz Alberto Duran Salomao wrote:
 
   Caros amigos: Este exercicio foi enviado para a lista jah faz algum
   tempo, mas parece-me
   que ainda nao foi respondida.
  
 
  [...]
 
  
   Um abraco a todos,
   Luiz Alberto Salomao
  
   Rubens Vilhena wrote:
  
   Olá, pessoal! Espero que me ajudem em minhas dúvidas sobre  Números
   Inteiros. 1) Se n é composto então o número 11111 (n vezes)
   também é composto. Obrigado!
  
  
  2,5 dúvidas. Primeira: a reçiproca é verdadeira? Ou seja, se
  111...11 (n vezes) é composto, então n tb é composto? Segunda: E com n
  primo? 111...11 (n vezes) é primo ou é composto? 2,5-ésima dúvida: E a
  reciproca (da segunda dúvida), é verdadeira?
 
  []'s
 
  Alexandre Tessarollo
 
  =
  Instruções para entrar na lista, sair da lista e usar a lista em
  http://www.mat.puc-rio.br/~nicolau/olimp/obm-l.html
  O administrador desta lista é [EMAIL PROTECTED]
  =

 =
 Instruções para entrar na lista, sair da lista e usar a lista em
 http://www.mat.puc-rio.br/~nicolau/olimp/obm-l.html
 O administrador desta lista é [EMAIL PROTECTED]
 =

=
Instruções para entrar na lista, sair da lista e usar a lista em
http://www.mat.puc-rio.br/~nicolau/olimp/obm-l.html
O administrador desta lista é [EMAIL PROTECTED]
=



Re: [obm-l] duvida

2002-04-10 Por tôpico Alexandre Tessarollo


Para simplifcar a vida, vou pensar numa cidade de 100 hab e vou
chamar
H (homens)/M (mulheres);
J (jovens com menos de 30)/I (idosos com +de 30);
S (solteiro)/C (casado).


 (unirio-rj) Tendo  sido feito o censo populacional 96 em uma cidade ,
descobriu-se sobre a população que:
1) 44% têm idade superior a 30 anos;
HIC+HIS+MIC+MIS=44
 2) 68% são homens ;
HJS+HJC+HIS+HIC=68
 3) 37% são homens com mais de 30 anos
HIC+HIS=37
 4) 25% são homens solteiros
HIS+HJS=25
 5) 4% são homens solteiros com mais de 30 anos
HIS=4
 6) 6%  são individuos  solteiros com mais de 30 anos
HIS+MIS=6
 com base nos dados anteriores ,pode-se afirmar q a porcentagem  da
população  desta  cidade q representa as mulheres casadas com idade
igual ou inferior a 30 anos é de?
MJC=?

Aplicando 5 em 3, 4 e 6, teremos, respectivamente: HIC=33; HJS=25 e
MIS=2. Aplicando esses resultados em 2, teremos HJC=6. Com isso já
calculamos todo o grupo dos homens e tb sabemos MIS. Com esses valores e
a equação 1, temos MIC=5. Hum... se somarmos td teremos q ter 100 (ou,
por 2, sabemos q só as mulheres somam 32), veremos que MJS+MJC=25. Bem,
aí não sei mais sair Creio q faltou algum informação... Alguém se
habilita? Abaixo os valores q encontrei, organizados.
 HJS = 25
HJC = 6
HIS = 4
HIC = 33
MJS = ?
MJC = 25-?
MIS = 2
MIC = 5

[]'s

Alexandre Tessarollo

=
Instruções para entrar na lista, sair da lista e usar a lista em
http://www.mat.puc-rio.br/~nicolau/olimp/obm-l.html
O administrador desta lista é [EMAIL PROTECTED]
=



Re: [obm-l] TN

2002-04-10 Por tôpico Alexandre Tessarollo



Luiz Alberto Duran Salomao wrote:

 Caros amigos: Este exercicio foi enviado para a lista jah faz algum
 tempo, mas parece-me
 que ainda nao foi respondida.


[...]



 Um abraco a todos,
 Luiz Alberto Salomao

 Rubens Vilhena wrote:

 Olá, pessoal! Espero que me ajudem em minhas dúvidas sobre  Números
 Inteiros. 1) Se n é composto então o número 11111 (n vezes)
 também é composto. Obrigado!


2,5 dúvidas. Primeira: a reçiproca é verdadeira? Ou seja, se
111...11 (n vezes) é composto, então n tb é composto? Segunda: E com n
primo? 111...11 (n vezes) é primo ou é composto? 2,5-ésima dúvida: E a
reciproca (da segunda dúvida), é verdadeira?

[]'s

Alexandre Tessarollo

=
Instruções para entrar na lista, sair da lista e usar a lista em
http://www.mat.puc-rio.br/~nicolau/olimp/obm-l.html
O administrador desta lista é [EMAIL PROTECTED]
=



[obm-l] Re: (a+bi)^(c+di)

2002-04-02 Por tôpico Alexandre Tessarollo



Agradeço às respostas sucintas do N e do Morgado e em particular, à prolixa
do JP :-)



Mas restou uma dúvida: se z=r*cis(t), então ln(z)=ln(r)+i*(t+2kpi). Foi
dito que 
z^w=e^(w*ln(z))=e^(c+d*i)*(ln(r)+i*(t+2kPI))=e^{[c*ln(r)-d*t-d*2kPI]+i[c*t-c*2kPI+d*ln(r)]}

Chamando X=c*ln(r)-d*t-d*2kPI e Y=c*t-c*2kPI+d*ln(r), temos 
z^w=e^(X+iY)=(e^X)*(e^iY)=(e^X)*((e^i)^Y)

Obviamente X e Y dependem do valor de k (do 2kPI). Mesmo assim, eu sei
calcular e^x. Só que eu não sei quanto vale e^i. Mesmo que seja um número
complexo, sei elevá-lo à Y, mas preciso saber como calcular e^i...

Imagino que deva sair pela série ou por outro caminho, mas nos meus rascunhos,
e^i pela série resulta em 
somatório [zero a infinito] {[16k^2+12k+1]*[4k+3+i]/(4k+3)!}

Como vcs podem ver, ficou meio feio... Como dizia o poeta, E agora, José?
(com todos os trocadilhos, JP :-


[]'s

Alexandre Tessarollo



=
Instruções para entrar na lista, sair da lista e usar a lista em
http://www.mat.puc-rio.br/~nicolau/olimp/obm-l.html
O administrador desta lista é [EMAIL PROTECTED]
=



Re: [obm-l] area do triângulo + erro

2002-03-31 Por tôpico Alexandre Tessarollo



Siberia Olympia wrote:

 Caros amigos, gostaria de ajuda na questâo

Seja ABCDE um pentágono de lados AB, BC, CD, DE e EA tal que
 Area(ABC)=Area(ABD)=Area(ACD)=Area(ADE)=17. Calcular a medida da área do
 triângulo BCE.

   Um forte abraço, cg.

Só por diversão, vamos construir a figura :-) Coloque A, B e C quaisquer
no plano. . Se ABC=ABD, então D está numa paralela à AB que passa por C ou no
reflexo desta paralela em relação à AB. Se ABD=ACD, então

D1 = C + vetor(AB)
D2 = C - vetor(AB)
D3 = intersecção da reta refletida com uma paralela à BC passando por A

Se ABD=ADE, então E está na paralela à AD passando por B ou no reflexo
desta em relação à AD. Como A, D2 e D3 estão alinhados, D2 e D3 dão o mesmo
par de retas para E: a reta Bc e o reflexo desta com relação à AD2 (AD3). Já
D1 dá uma passando por C e outra por B, mas como ADB=ADC, então essas
paralelas são equidistantes de AD1, logo não há problemas.

Portanto, temos dados A, B e C, temos 3 opções para D, depois disso,
colocar o E em qualquer lugar de duas paralelas à AD.


Se escolhermos D2 ou D3, não poderemos colocar E sobre a reta BC ou
área(BCE)=0. Assim, coloquemos sobre o reflexo de BC em relação à AD2 (AD3).
área(BCE) = BC*(dist[E,BC])
Como A equidista da reta em que está E e da reta BC, temos que
dist[E,BC]=2*dist[A,BC]. Logo:
área(BCE) = 2*BC*(dist[A,BC]) = 2*área(ABC) = 34


No entanto, se escolhermos D1, a área de BCE poderá variar de zero a
infinito. Alguém sabe explicar qual o erro em D1?? Tudo leva a crer que D2 e
D3 estejam certos, mas qual o erro de D1? Na figura é fácil ver que o único
ponto E que dá a mesma resposta é (a intersecção de CD1 com a paralela à AD1
que passa por B) ou, equivalentemente, (D1+vetor(AB)). Ah sim, tb há o
reflexo deste ponto em relação à BC. E aí, alguém se habilita??


[]'s

Alexandre Tessarollo

=
Instruções para entrar na lista, sair da lista e usar a lista em
http://www.mat.puc-rio.br/~nicolau/olimp/obm-l.html
O administrador desta lista é [EMAIL PROTECTED]
=



[obm-l] (a+bi)^(c+di)

2002-03-30 Por tôpico Alexandre Tessarollo

Esta é para todos, em especial o JP e o séquito de seguidores que se forma
aqui na lista :-)

  Como faço para elevar um número complexo qualquer a outro? Sei que isso
é possível, mas ainda não vi essa parte na fac... Seria possível pelo menos
uma dica ou idéia de como se faz? Alguma bibliografia?

[]´s e Feliz Páscoa!!

Alexandre Tessarollo



=
Instruções para entrar na lista, sair da lista e usar a lista em
http://www.mat.puc-rio.br/~nicolau/olimp/obm-l.html
O administrador desta lista é [EMAIL PROTECTED]
=



Re: [obm-l] Novov grupo

2002-03-26 Por tôpico Alexandre Tessarollo



[EMAIL PROTECTED] wrote:

 Fiquei sabendo de um novo grupo de discussões que é muito bom e trata de
 todos os assuntos relacionados às exatas, indo desde a Matemática superior
 até a Engenharia em geral.

Bem, ia perguntar qual o endereço desse grupo, mas vc já deu (em outra
msg) :-))

Notei que vc mandou essa msg não só p/a OBM-1 como para um
[EMAIL PROTECTED]. Gostaria de saber mais sobre esse grupo
tb. Depois manda p/a lista (OBM) as instruções p/inscrição e/ou uma home-page
com as descrições de cada lista, por favor.



 Vale a pena!
 Um abraço!

[]'s

Alexandre Tessarollo


=
Instruções para entrar na lista, sair da lista e usar a lista em
http://www.mat.puc-rio.br/~nicolau/olimp/obm-l.html
O administrador desta lista é [EMAIL PROTECTED]
=



Re: [obm-l] Álgebra

2002-03-22 Por tôpico Alexandre Tessarollo


Hum... 133^5 termina em 3; 110^5 termina em 0; 84^5 termina em 4; 27^5
termina em 7; logo, n^5 termina em 3+0+4+7 q termina em 4. Portanto, o único
algarismo p/a unidade de n é 4 - basta ver q SÓ os números terminados em 4,
qundo elevados a 5, terminam em 4. Tente ver se sai agora... Se não me engano
tinha um truque de Ágebra I que toernava este prob MUITO fácil, mas não
estou lembrando agora. Se eu achar antes de alguém aqui responder eu mando.

[]'s

Alexandre Tessarollo

Rafael WC wrote:

 Olá Pessoal!

 Gostaria de saber uma resolução para esse exercício:
 O inteiro positivo n, tal que
 133^5 + 110^5 + 84^5 + 27^5 = n^5

 é igual a...

 A resposta é n = 144, mas e a resolução???

 Obrigado,

 Rafael.

 =
 Rafael Werneck Cinoto
   ICQ# 107011599
 [EMAIL PROTECTED]
   [EMAIL PROTECTED]
   [EMAIL PROTECTED]
 http://www.rwcinoto.hpg.com.br/

 __
 Do You Yahoo!?
 Yahoo! Movies - coverage of the 74th Academy Awards®
 http://movies.yahoo.com/
 =
 Instruções para entrar na lista, sair da lista e usar a lista em
 http://www.mat.puc-rio.br/~nicolau/olimp/obm-l.html
 O administrador desta lista é [EMAIL PROTECTED]
 =


=
Instruções para entrar na lista, sair da lista e usar a lista em
http://www.mat.puc-rio.br/~nicolau/olimp/obm-l.html
O administrador desta lista é [EMAIL PROTECTED]
=



Re: [obm-l] Problema de Geometria

2002-03-10 Por tôpico Alexandre Tessarollo



Mensagem original

Calcular a área de um triângulo retângulo de perímetro 2p e altura relativa
a hipotenusa h.

=
Instruções para entrar na lista, sair da lista e usar a lista em
http://www.mat.puc-rio.br/~nicolau/olimp/obm-l.html
O administrador desta lista é [EMAIL PROTECTED]
=



   Hum... Bem, por preguiça vou chamar o perímetro de P mesmo... a é a
hipotenusa.

a+b+c=P
b+c=P+a
b^2+c^2+2bc=P^2+a^2-2aP(I)

a^2=b^2+c^2(II)

ah=bc  (III)

Aplicando II em I, temos:
a^2+2bc=P^2+a^2-2aP
2bc=P^2-2aP

Aplicando III, temos
2ah=P^2-2aP
a=P^2/(2(h-P)) (IV)

Note que II pode ser reescrita como
a^2=(b+c)^2-2bc

Aplicando III, temos
a^2=(b+c)^2-2ah
2ah=(b+c)^2-a^2
2ah=(b+c+a)(b+c-a)

Lembrando que P=a+b+c, temos
2ah=P(P-2a)

Lembre que a área S procurada pode ser escrita como S=ah/2. Assim, temos:
4S=P(P-2a)

Aplicando IV, temos:
4S=P(P-2(P^2/(2(h-P
4S=P(P-(P^2/(h-P)))
4S=P(P+P^2/(P-h))

Rearrumando para ficar mais simpático:

4S=P(2P^2-hP)/(P-h)
S=P(2P^2-hP)/(4(P-h))

Se eu não errei nenhuma conta, é isso aí... Espero ter ajudado...

[]'s

Alexandre Tessarollo




=
Instruções para entrar na lista, sair da lista e usar a lista em
http://www.mat.puc-rio.br/~nicolau/olimp/obm-l.html
O administrador desta lista é [EMAIL PROTECTED]
=



Re: [obm-l] Angulo de 1 grau

2002-03-09 Por tôpico Alexandre Tessarollo



Jose Paulo Carneiro wrote:

 Se 1 grau fosse construtivel, entao 20 graus tambem seria (facil).
 No entanto, 20 graus nao eh construtivel, como estah em todos os livros
 (por exemplo, Construcoes geometricas, de E.Wagner, apendice).
 JP

Não sei se me expressei bem, mas o problema de 19 me fez pensar no de 1.
Quis colocar um problema SEPARADO do de 19, OUTRO problema, OUTRA discussão.
O de 19 serviu apenas p/me inspirar.. :-))

Respondendo à resposta do Nicolau:

Vou olhar com calma a sua resposta, mas gostaria de obter algo mais
geométrico, se possível, puramente geométrico, apenas régua e compasso. Se
for o caso, vale incluir lugares geométricos construtíveis ponto-a-ponto,
como uma hipérbole, por exemplo. Afinal, Cabri, SketchPad e similares estão
aí p/isso  :-)

(OBS: Quem já conhece, saiba que o SkewtchPad 4 já possui uma versão demo
disponível p/downloads em www.keypress.com/sketchpad)

[]'s

Alexandre Tessarollo



 - Original Message -
 From: Alexandre Tessarollo [EMAIL PROTECTED]
 To: [EMAIL PROTECTED]
 Sent: Saturday, March 09, 2002 2:27 AM
 Subject: [obm-l] Angulo de 1 grau

 Estava olhando um prob que propuseram na lista (DADO um ângulo de
 19, construir o de 1) e lembrei que o ângulo de 1 não é construtível,
 mas não lembro como provar... Alguém se habilita?

 Além disso, quais são os ângulos  construtíveis por régua e
 compasso? Refiro-me aos ângulos de valores inteiros em graus, como por
 exemplo 15, 30, 45, 60

 Para finalizar, sei que podemos bissectar um ângulo mas não
 trisectá-lo. É possível dividí-lo em n partes, n NÃO sendo uma potência
 de 2?

 Aliás, na faculdade um professora minha até me mostrou uma maneira
 de trissectar usando régua ESCALONADA e compasso. Mas foi só um
 comentário rápido e já se vai um bom tempo... Alguém?

 []'s

 Alexandre Tessarollo

 =
 Instruções para entrar na lista, sair da lista e usar a lista em
 http://www.mat.puc-rio.br/~nicolau/olimp/obm-l.html
 O administrador desta lista é [EMAIL PROTECTED]
 =

 =
 Instruções para entrar na lista, sair da lista e usar a lista em
 http://www.mat.puc-rio.br/~nicolau/olimp/obm-l.html
 O administrador desta lista é [EMAIL PROTECTED]
 =


=
Instruções para entrar na lista, sair da lista e usar a lista em
http://www.mat.puc-rio.br/~nicolau/olimp/obm-l.html
O administrador desta lista é [EMAIL PROTECTED]
=



Re: [obm-l] Provar q o produto de 3 nr consecutivos...

2002-03-08 Por tôpico Alexandre Tessarollo


Hum... Bem, se são 3 números consecutivos, então ou o segundo é par ou os
outros dois são. Produto de um par opor qualquer número inteiro é sempre par.
(No fundo, é a mesma explicação do Hugo, mas sem conta :-))

[]'s

Alexandre Tessarollo

Hugo Iver Vasconcelos Goncalves wrote:

 (x-1)*x*(x+1)
 x^3 - x

 sejam os numeros pares da forma 2k e os ímpares da forma 2k+1.

 se x par então:
 (2k)^3 - 2k
 2(4k^3 - k) .

 se x ímpar então:
 (2k + 1)^3 - (2k+1)
 2(4k^3 + 6k^2 +2k) .

 []´s hugo
 - Original Message -
 From: DC [EMAIL PROTECTED]
 To: [EMAIL PROTECTED]
 Sent: Friday, March 08, 2002 2:03 PM
 Subject: [obm-l] Provar q o produto de 3 nr consecutivos...

 Oi pessoal,

   Preciso de uma ajuda:
 Prove que o produto de 3 numeros inteiros consecutivos
 é par.

   Amplexos,

   Douglas Fabiano Drumond de Carvalho
 
 UAI - ESTADO DE MINAS
 O Grande portal dos Mineiros na Internet
 http://www.uai.com.br
 =
 Instruções para entrar na lista, sair da lista e usar a lista em
 http://www.mat.puc-rio.br/~nicolau/olimp/obm-l.html
 O administrador desta lista é [EMAIL PROTECTED]
 =

 =
 Instruções para entrar na lista, sair da lista e usar a lista em
 http://www.mat.puc-rio.br/~nicolau/olimp/obm-l.html
 O administrador desta lista é [EMAIL PROTECTED]
 =


=
Instruções para entrar na lista, sair da lista e usar a lista em
http://www.mat.puc-rio.br/~nicolau/olimp/obm-l.html
O administrador desta lista é [EMAIL PROTECTED]
=



[obm-l] Angulo de 1 grau

2002-03-08 Por tôpico Alexandre Tessarollo



Estava olhando um prob que propuseram na lista (DADO um ângulo de
19, construir o de 1) e lembrei que o ângulo de 1 não é construtível,
mas não lembro como provar... Alguém se habilita?

Além disso, quais são os ângulos  construtíveis por régua e
compasso? Refiro-me aos ângulos de valores inteiros em graus, como por
exemplo 15, 30, 45, 60

Para finalizar, sei que podemos bissectar um ângulo mas não
trisectá-lo. É possível dividí-lo em n partes, n NÃO sendo uma potência
de 2?

Aliás, na faculdade um professora minha até me mostrou uma maneira
de trissectar usando régua ESCALONADA e compasso. Mas foi só um
comentário rápido e já se vai um bom tempo... Alguém?

[]'s

Alexandre Tessarollo


=
Instruções para entrar na lista, sair da lista e usar a lista em
http://www.mat.puc-rio.br/~nicolau/olimp/obm-l.html
O administrador desta lista é [EMAIL PROTECTED]
=



Re: [obm-l] Sebos

2002-02-20 Por tôpico Alexandre Tessarollo

Eduardo Quintas wrote:

 Olá pessoal
 Gostaria de saber endereços de bons sebos na cidade do rio de janeiro...

A Livraria São José, no Centro, é uma ótima. Uns tempos atrás recebeu
a
coleção de um porfessor da Uerj(?) que havia falecido. Não estou me lembrando
do
endereço agora, mas depois ponho aqui na lista. Se alguém lembrar antes...



 Estou procurando o livro do caronet de áreas - tomo iv e não sei onde
posso
 conseguir será que tem na biblioteca do impa ?

A biblioteca do Impa está online. Basta ir ao site www.impa.br e seguir
o
link biblioteca. Lá vc poderá procurar dentre os títulos do Impa - a melhor
biblioteca em pós-graduação do RJ. A segunda melhor é a da UFRJ. Seu acervo
pode
ser consultado no site www.minerva.ufrj.br e procurar no acervo geral.



 o livro Porblemas Selecionados de Matemática - do Agostino eu tenho o
vol. 1
 (Fundamentos de álgebra e análise) - ao todo são quantos volumes ? de
que se
 tratam esses volumes ? será que é complicado consegui-los ? mesmo xerox


Se você estiver falando do livro de Raul Agostino, ele já foi comentado
aqui
na lista. Vale lembrar que o autor comentou comigo que NÃO pretende dar
continuidade ao projeto. Consulte o arquivo (da lista) para maiores detalhes.

[]'s

Alexandre Tessarollo

PS: Aqueles que quiserem, tenho um cópia xerox que tirei do exemplar que
o Raul
tem (ele só tem essa cópia, mais nenhuma). Quem quiser, só entrar em contato
que
dá-se um jetio de tirar um cópia dessa minha cópia. Em respeito aos outros
participantes da lista, peço que usem meu e-mail pessoal [EMAIL PROTECTED]




=
Instruções para entrar na lista, sair da lista e usar a lista em
http://www.mat.puc-rio.br/~nicolau/olimp/obm-l.html
O administrador desta lista é [EMAIL PROTECTED]
=



Re: [obm-l] Espirais da Montanha Russa

2002-01-31 Por tôpico Alexandre Tessarollo

Rogério da Silva Ignácio wrote:

  prezados participantes da lista.

  Preciso plotar uma simulação de uma montanha russa em 2D que tenha
pelo menos
  dois loopings.

  Pesquisei sobre as espirais usadas nos loopings das montanhas russas e

  encontrei referência a dois tipos:

  i  Em forma de parafuso (complicado para potar em 2D).
  ii Uma espiral em forma de gota (razão de ser desta mensagem).

  Bom, pesquisei o quanto pude sobre essa espiral a fim de saber seu
nome e sua
  parametrização. Encontrei apenas uma referência na revista
superinteressante de
  janeiro de 89. Nesta revista esplica-se que não se utiliza o formato
circular
  para o looping por razões físicas e biológicas pois permite pontos de
intensa
  gravidade e outros de nula. Então, segundo a revista, a solução foi
encontrada
  em 1977 numa curva chamada Espiral de cornu ou clotóide atribuída a
Euler.
  Porém não vejo nesta curva ( de paramétricas dy/dt=sen(t^2)
dx/dt=cos(t^2) )
  solução para o meu problema pois preciso plotá-la para ser vista de
lado e
  não de frente e o software que tenho que usar
  (http://phoenix.sce.fct.unl.pt/modellus) não possui recursos 3D.

  Alguém conhece alguma espiral que me sirva para esse fim? Ou como
plotar a
  clotóide em outra perspectiva?
  Agradeço antecipadamente
  Rogério

=

  Instruções para entrar na lista, sair da lista e usar a lista em
  http://www.mat.puc-rio.br/~nicolau/olimp/obm-l.html
  O administrador desta lista é [EMAIL PROTECTED]

=




Hum... Bem, resolvi partir para ignorância e pedi pro Maple
integrar as paramétricas que vc mostrou e depois
plotei. Como eu trabalho com curvas, reconheci a figura e consegui achar

a referência que eu tinha: Enciclopédia
Mirador de 1995, no vocábulo curvas. Acredite ou não, não encontrei
nenhum outro livro de curvas planas que
mostrasse essa e algumas outras curvas... Segue o parágrafo da clotóide:

Clotóide ou espiral de Euler (ou espiral de Cornu ou espiral de
Fresnel). É a curva de equações paramétricas

x = (a/sqrt(2)) * int[0,t]((sen(t)/sqrt(t))dt)
y = (a/sqrt(2)) * int[0,t]((cos(t)/sqrt(t))dt)

possuindo dois pontos assintóticos, de coordenadas (a*sqrt(pi/8),
a*sqrt(pi/8)) e (-a*sqrt(pi/8), -a*sqrt(pi/8)). Foi
inicialmente estudada por Euler e se originou de problemas relacionados
com molas; mais tarde teve sua importância
aumentada devido aos trabalhos de Fresnel sobre difração da luz.

Creio que não é permitido mandar arquivos attachados (Nicolau, que
tal escrever essas regrinhas básicas no
rodapé?), então vou mandar o arquivo diretamente para vc - são só 4k.

[]'s

Alexandre Tessarollo

OBS: int[a,b]((f)dx)  é a integral da função f em relação à variável x
no intervalo de a a b.

OBS: Desculpem a aparente demora, tive problemas na configuração dos
programas de e-mail e a lista estava negando as minhas msgs... Creio que
agora eu consegui resolver...

=
Instruções para entrar na lista, sair da lista e usar a lista em
http://www.mat.puc-rio.br/~nicolau/olimp/obm-l.html
O administrador desta lista é [EMAIL PROTECTED]
=



Re: 2 questões...

2001-12-13 Por tôpico Alexandre Tessarollo



henrique.vitorio wrote:
 
 
 Oi,
 Saudações a todos,meu nome eh Henrique(sow de
 Recife) e sow novo nessa lista.Entaum..aí vaum umas
 questões que gostaria que me ajudassem.

Bem-vindo...

   1- encontre todas soluções inteiras positivas de:
7^(x) + 1 = 5^(z) + 3^(y) (nessa questão soh consegui
 mostrar que x,y e z têm que ser ímpar).
   2-Sabe-se que os vértices de um triângulo pertencem à
 hipérbole xy=1.Prove que seu ortocentro também pertence
 a essa hipérbole.


Bem, sejam A(a,1/a), B(b,1/b) e C(c,1/c) os vértices do triângulo e
suas respectivas coordenadas. Os coeficientes angulares das retas AB e
AC são -1/ab e -1/ac respectivamente. Assim, as alturas relativas a
esses lados terão coeficintes angulares ab e ac, respectivamente.
Completando as equações dessas alturas, ficam, respectivamente:
y = abx + 1/c - abc
y = acx + 1/b - abc

Para achar o ortocentro, basta resolver o sistema. encontraremos
x=-1/abc e y=-abc, logo, xy=1, CQD...

Sei que não expliquei tão bem quanto deveria, mas tenho o péssimo
hábito de checar e responder e-mails de madrugada. De qualquer forma,
usei o fato de que os três vértices estavam sobre a hipérbole xy=1
quando disse que A era (a, 1/a), por exemplo. Depois tirei a equação de
dois lados, das perpendiculares a esses lados passando pelo vértice
oposto e fiquei com duas equações. Como sabia que o único ponto em comum
era o ortocentro, bastou resolver o sistema e pronto. Para concluir,
bastou notar que, para quaisquer A, B e C na hipérbole xy=1, o
ortocentro de ABC também estará nessa hipérbole, C.Q.D.

 
  Desde jah agradeçoFalow!
  Henrique

[]'s

Alexandre Tessarollo



Re: somatorio

2001-12-04 Por tôpico Alexandre Tessarollo



Gustavo Nunes Martins wrote:
 
 Ha uma formula que diz que o a soma dos numero de uma PA que comeca com
 o numero n1 e acaba com o numero nx e
 (n1+nx)x/2. Um jeito muito simples de se descobrir esta formula pode ser
 econtrado em http://galileu.globo.com/edic/112/eureca.htm.
 

Aliás, esse é o único jeito q conheço de DEDUZIR esta fórmula. Até é
possível PROVÁ-LA com indução, mas deduzir acho q só assim mesmo

 Como acho a expressao que me da a soma dos numeros da sequencia
 n^2 + (n+1)^2 + (n+2)^2 + (n+3)^2 + ... ?

Hum... Vc está querendo somar vários quadrados perfeitos consecutivos a
partir de um n^2 qualquer, certo? Pois bem, vamos passo a passo pq já
são passa das duas e meus neurônios foram dormir :-)

n^2 + (n+1)^2 + (n+2)^2 + (n+3)^2 + ... + (n+k)^2 =
n^2 + (n^2+2n+1) + (n^2+4n+4) + (n^2+6n+9) + ... + (n^2+2kn+k^2) =

Olhando com carinho, vemos um total de k+1 termos n^2. Note também
que ops termos de primeiros grau podem ser arrumados de forma sugestiva
coma a abaixo. Por fim, restam os termos independentes. Vejamos como
fica...

(k+1)n^2 + (2+4+6+...+2k)n + (1+4+9+...k^2)

Fazendo por partes:
I) (k+1)n^2
É, acho q melhor q isso não fica, hehehe :-)

II)(2+4+6+...+2k)n
é uma PA. Assim, fica
k(k+1)n/2

III)(1+4+9+...k^2)
é a soma de todos os quadrados perfeitos a partir do primeiro (zero ou
um, tanto faz..). Se vc preferir, um caso particular do seu prob.
Observando com cuidado , temos:

4-1=3
9-4=5   5-3=2
16-9=7  7-5=2
25-16=9 9-7=2
.
.
.
Note que os quadrados não formam a PA q vc conhece, mas a diferença
deles sim. Isso significa q os quadrados pefeitos formam uma PA de
segunda ordem, pq só na segunda vez q calculamos a diferença é q
chegamos a valores iguais. A PA q vc conhece é chamada de PA de
primeira ordem, porque já na primeira vez q calculamos a diferença
chegamos a valores iguais. Podemos ter PAs de qualquer ordem

Mas, voltando ao nosso problema, temos q descobrir como somar essa tal
de PA de segunda ordem. Como já são duas e meia, deixo p/vc tentar um
pouco. Sugiro tentar estabelecer alguma fórmula de recorrência, depois
uma do termo geral e, por fim, a da soma. Vale lembra q essas fórmulas,
na PA, são, respectivamente:
a[n] = a[n-1] + R
a[n] = a[1] + (n-1)R
S[n]=(a[1]+a[n])n/2

OBS: a[n] indica n-ésimo termo da seqüencia

Depois q você souber somar esta PA de segunda ordem basta somar I, II e
III q vc terá seu resultado... Se eu conseguir tempo amanhã eu faço o
resto...

[]'s

Alexandre Tessarollo



Re: Um quadrado repartido

2001-11-28 Por tôpico Alexandre Tessarollo



Ricardo Miranda wrote:
 
 Olá. Alguem pode me ajudar a encontrar a área do triângulo formado
 interno ao quadrado na figura anexa?
 E quanto à medida das semi-retas que cortam o quadrado (as bases
 menores dos trapézios formados, que sao iguais aos lados do triângulo)?
 
 Agradeço a ajuda,
 
  Ricardo Miranda M
 [EMAIL PROTECTED]
 

Hum, na sua figura eu vou começar chamando de A o vértice superior
esquerdo, B C e D os vértices seguintes. M é o ponto médio de AB e P é o
ponto dentro de ABCD. Fazendo Pitágoras em AMD e BMC temos q
MD=MC=sqrt(5)/2. Prolongue MP até o lado CD e determine um ponto Q em
CD. Olhando para o triângulo retângulo MQC temo MQ=1, QC=1/2 e
MC=sqrt(5)/2. Logo, cos(QMC)=2/sqrt(5).  Seja MP=PC=x. Aplicando lei dos
cossenos em M no triângulo MPC, temos x=5/8, se eu não errei conta
alguma.

Agora, com tudo isso na mão, fica fácil. A segunda questão já foi
respondida, é x=5/8. A primeira, basta aplicar Heron ou o seu método
preferido e pronto: área de PCD = 3sqrt(2)/32.

Afora ewventuais erros de conta, acredito q tudo esteja certo...
Comentem...

[]'s

Alexandre Tessarollo



Re: 2 de geometria

2001-11-27 Por tôpico Alexandre Tessarollo

Aqueles que quiserem uma figurinha, podem me pedir, eu tenho aqui. Pela
recente mesngame do volume de óleo já vi q é possível mandar anexos, mas
antes gostaria de saber qual a política oficial da lista com respeito a
anexos. Nicolau, por favor...

Alexandre Tessarollo wrote:
 
 Ao povo q gosta de gemetria, seguem duas. A primeira foi um aluno que
 me passou, tirada de um livro de segundo grau das antigas (Exercícios
 de Gemetria Plana, do Edgar de Alencar Filho). A outra acho q até já
 postei aqui, não tenho certeza. Mas tirei de outra lista. Vamos a elas:
 
 1) Por um ponto P interno a um triângulo ABC traçam-se a rretas
 paralelas aos seus lados, que o decompõem em seis partes, três das quais
 triângulos de área S[1], S[2] e S[3]. Achar a área do triângulo ABC.
 
 Eu até tenho a respoata, mas ainda não sei como chegar nela.
 
 2) Tome um pentágomno qualquer ABCDE e as retas suportes dos lados. Note
 que elas detrminam um triângulo em cima de cada lado do pentágono.
 Construa as circunferências circunscritas a esse triângulos. Note que as
 circunferências de lados adjacentes se interceptam duas a duas em dois
 pontos: um dos vértices do pentágono e outro. Chamemos esses outros
 pontos convenietemente de A', B', C', D' e E'. Prove que A', B', C', D'
 e E' pertencem a uma mesma circunferência.
 
 Não lembro de qual das n! listas que tirei esta questão, mas lembro q
 mencionava uma relação não confirmada a algum político chinês recente.
 Não sei se teria sido este chinês a formular ou se foi devido a ele que
 o prob ficou conhecido. Seria uma caso similar ao problema do cavalo do
 presidente, né Nicolau? :0)))
 
 []'s
 
 Alexandre Tessarollo



Re: Um tal de Newton...

2001-11-21 Por tôpico Alexandre Tessarollo



Alexandre F. Terezan wrote:
 
 Uma pequena distracao:
 
 (1 + 3x + 2x^2) = 2(x+1)(x+1/2)
 e nao
 (1 + 3x + 2x^2) = (x+1)(x+1/2)
 


Razões pelas quais pessoas como eu não deveriam mandar e-mails às 2:40
da manhã... Não sei pq ainda insisto... hehehe

[]'s

Alexandre Tessarollo

 -Mensagem Original-
 De: Alexandre Tessarollo [EMAIL PROTECTED]
 Para: [EMAIL PROTECTED]
 Enviada em: Quarta-feira, 21 de Novembro de 2001 02:41 Terezan
 Assunto: Re: Um tal de Newton...
 
 [EMAIL PROTECTED] wrote:
 
  Meus cumprimentos,
 
  Estava estudando um tal de Newton e encontrei uma questão
  interessante, embora eu esteja errando algo simples pra vocês...
 
  Questão (FFCLUSP)
  Mostrar que o coeficiente de x^8 no desenvolvimento
  de (1 + 3x + 2x^2)^10 é 3780.
 
  Meu erro: os coeficientes de x e de x^2 estão fazendo o
  coeficiente do termo x^8 ficar muito grande ...
 
 Vale lembrar q se vc estiver tentando usar a fórmula do Binômio de
 Newton, ela só vale para BInômios, ou seja, algo como (x+a)^n. Nós temos
 um TRInômio...
 
 Bem, mas vamos tentar... Sabemos q o trinômio pode ser reescrito como
 (x+1)(x+1/2). Assim, queremos saber o coeficiente de x^8 no
 desenvolvimento de
 [(x+1)(x+1/2)]^10= [(x+1)^10][(x+1/2)^10]
 
 Seja a[i]x^i o termo de grau i do primeiro binômio e, p/não
 confunidir as letras, a[j]x^j o de grau j do segundo binômio. Assim, o
 nosso polinômio final terá termos da forma a[i]a[j]x^(i+j), com i e
 j variando (independentemente) de 0 a 10.
 
 Dessa forma, temos que achar i+j=8. As soluções (i;j) que estão no
 nosso intervalo são: (0;8), (1;7), (2;6), (3;5), (4;4), (5;3), (6;2),
 (7;1) e (8;0). Agora sim podemos utilizar a fórmula do Binômio de
 Newton, calcular os coeficientes com i e j das soluções, fazer as
 devidas contas e pronto. Sei q deve dar algum trabalho, mas depois posso
 até fazer caso alguém queira. Como a essa hora meus neurônios já foram
 dormir, fico devendo uma solução mais concisa e prática.
 
 []'s
 
 Alexandre Tessarollo
 
  Caso alguém queira tentar...
 
  Muito grato,
 
  Héduin Ravell
 
  _
  Do You Yahoo!?
  Get your free @yahoo.com address at http://mail.yahoo.com



Re: Somatórios

2001-11-20 Por tôpico Alexandre Tessarollo

Bruno Furlan

Nunca estudei limite de soma, mas vou tentar:
 
Sum[(x³+13)/2n] = (x³+13).Sum(1/2n) = (x³+13)/2.Sum(1/n).
Como o limite de Sum(1/n) é 2, temos que Sum[(x³+13)/2n] = x³+13.

Concordo com tudo antes e tudo depois dessa linha. Só não concordo quan
vc diz que o limite da soma de 1/n com n variando de 1 a infinito dá 2.
A soma da PG (1/2)^n com n variando de zero (só p/somar 1) a infinito é
que dá 2. O somatório conforme apresentado representa a série harmônica
- a primeira que se estuda quando se aprende séries de verdade. Creio
inclusive que este tema já foi discutido aqui... Se alguém se interessar
há uma prova bem intuitiva e convincente.

[]'s

Alexandre Tessarollo



Re: dúvida

2001-11-14 Por tôpico Alexandre Tessarollo



Arnaldo wrote:

 
 Um automóvel comporta dois passageiros nos bancos da frente e três no detrás.

 Calcule o número de alternativas distintas para lotar o automóvel com pessoas

 escolhidas dentre sete, de modo que uma dessas pessoas nunca ocupe um lugar

 nos bancos da frente
 
 Vamos chamar de (A) a pessoa que nunca pode ocupar o banco da frente. Quando
 (A) ocupa um dos bancos de trás sobram 6 pessoas para ocupar os outros quatro
 bancos e portanto temos arranjo(6,4) = 360 maneiras distintas,mas como (A) pode
 acupar três bancos então o total de maneiras distintas são 3*360 = 1080.


Hum... Praticamente correto. Se o cidadão estiver no carro, então realmente
teremos 1080 maneirs de arrumar todo mundo. Mas ele pode ficar de fora, por quê
não? Nesse caso, teríamos 6 pessoas para distribuir em 5 lugares. Fazendo as
contas, temos 720 maneiras. Somando os dois casos (com A e sem A), temos
1080+720=1800.

[]'s

Alexandre Tessarollo

PS: O povo daqui não vai comentar a prova do IME deste ano não? Estava
interessantíssima, especialmente se tentarmos resolvê-la APENAS com conteúdo de 2º
grau/Ensino Médio...




IME (era: Re:dúvida)

2001-11-14 Por tôpico Alexandre Tessarollo



luis felipe wrote:

 concordo com o alexandre

 a prova do IME deste ano foi bem elaborada, embora eu ache que duas questões
 estavem pesadas demais para alunos de 2 grau( 7 e a 9) devemos lamentar
 também uma falha grave no enunciado da questão 8

 valeu

 luis felipe

Já que ninguém comenta, comento eu. Comecemos pela questão 9.

Resolva a equação sqrt(5-sqrt(5-x))=x sabendo-se que x0.

Eu já devo ter visto umas 4 soluções diferentes, mas em quase todas havia
pelo menos um passo não justificado ou questionável... Uma delas era:

Seja f(x) = sqrt(5-x). Temos f(f(x))=x. Logo,  f(x)=f^(-1)(x). Aí vem a parte é
fácil ver que os gráficos de f(x) e de f^(-1)(x) se cruzam sobre a reta y=x. A
partir daí, temos f(x)=x, resolve-se uma equação do segundo grau e pronto. Mas
falta demonstrar a parte é fácil ver...

Outra diz:

Aplicando f(x) nela mesma 2n vezes, com n tendendo ao infinito, teremos
f(f(f(f((f(x))...=x. Logo, podemos trocar todos os f(f(f...(f(x))...)))
de dentro do primeiro f por x. Assim teremos f(x)=x e novamente é só
resolver a eq do segundo grau. A solução, olhando com carinho, está certa, mas
foi utilizado o conceito de limite.

Ainda há uma terceira, esta já sem erros mas um pouco mais longa. Trata-se da
solução do Poliedro:

Como x0 e real, temos que 0x5. Tome y=sqrt(5-x) (I). A equação original
transforma-se em
sqrt(5-y)=x (II)
Elevando I e II ao quadrado, temos:
y^2=5-xIII
x^2=5-yIV
Fazendo III-IV, temos
y^2-x^2=y-x
(y+x)(y-x)=y-x
(y+x)(y-x)-(y-x)=0
(y-x)(y+x-1)=0

Segue que
y-x=0V

OU

y+x-1=0VI

De V segue a nossa equação do segundo grau. Considerando o intervalo 0x5,
só teremos uma resposta - a certa. Falta examinar VI. Substituindo-a em III ou
IV, teremos uma equação do segundo grau que resulta só uma resposta no intervalo
0x5. Contudo, como elevamos ao quadrado as eqs I e II p/chegarmos a III e IV,
precisamos verificar via teste se essas duas soluções servem ou não. Fazendo
isso só teremos a resposta correta...

Há ainda uma resposta, esta feita pelo Prof. Raul Agostino:

Elevando a equação ao quadrado e arrumando, temos:
5-x=sqrt(5-x)

Elevando novamente, temos:

25-10x^2+x^2=5-x

O que todo mundo tenta daqui em diante é somar tudo num lado só, chegar num
polinômio do QUARTO grau e não conseguir resolvê-lo - ele não possui raízes
óbvias, sequer inteiras... O pulo do gato segue abaixo, se vc não quiser ver,
pare aqui..

10

9

8

7

6

5

4

3

2

1

0

Bum!! Brincadeirinha... :0)

Olhando com MUITO carinho e MUITA boa vontade, podemos arrumar a equação
assim:

25-(2x^2+1)5+x^4+x=0

Um olho treinado verá uma equação do SEGUNDO grau em CINCO. Isso mesmo, algo
da forma a(5^2)+b(5)+c=0. Resolvendo, teremos:

5=(2x^2+1 +-sqrt(4x^4+4x^2+1-4x^4-4x))/2
Dentro da raiz fica 4x^2+1-4x = (2x-1)^2. Tirando a raiz, deveríamos colocar o
módulo mas, como já existe o +-, basta colocar direto mesmo. Fica:

5=(2x^2+1 +-(2x-1))/2

Resolvendo e respeitando os intervalos, teremos a solução...

[]'s

Alexandre Tessarollo




Re: RES: ajuda em um problema (oops)

2001-10-11 Por tôpico Alexandre Tessarollo


[...]

...esse quadrilátero [MNPQ] é _circunscritível_ e I é seu centro, já que
ele equidista dos lados... 

Oops!!! Minhas mais sinceras desculpas!! Pessoas como eu não deviam ler
nem tampouco responder e-mails de amdrugada... Tinha lido incritível
ao invés de circunscritível... Minhas mais sinceras desculpas ao
Einstein e ao Harold... Tanto a questão como a resposta estão
corretíssimas, e isso eu digo agora às 15:20h, em sã consciência!! :-))

Bem, de qualquer forma aproveito para lançar outra questão: Seja ABCDE
um pentágono QUALQUER. Prolongue os lados de forma que eles se encontrem
dois a dois, determinando os triângulos FAB, GBC, HCD, IDE, JEA sendo F,
G, H, I e J os pontos de intersecção (Agora a figura tornou-se um
pentágono estrelado). Construa os círculos circunscritos a estes
triâgulos e note que eles se interceptam em dois pontos dois a dois.
Cinco pontos são A, B, C, D, e E e os outros cinco chamemos
convenientemente de A', B', C', D' e E'. Prove que A'B'C'D'E' é
circuncritível para qualquer ABCDE (convexo?).

[]'s

Alexandre Tessarollo



Re: RES: ajuda em um problema

2001-10-09 Por tôpico Alexandre Tessarollo

Solução:
Faz a figura para ficar mais fácil de ver...
Como M, N, P e Q são as projeções e I  sobre  os lados AB, BC, CD, DA
temos que:
Os quadriláteros BMNI, NIPC, PIQD, MIQA são todos inscritíveis já que
possuem angulos opostos somando 180 graus. 
Como o quadrilátero ABCD é inscritível, temos que: CBD=CAD=x (ler
angulo BCD...)
ABD=ACD=y, BAD=BDC=z, ADB=BCA=w. 
Como:
BMNI é inscritível: MBI=MNI=y, NBI=NMI=x  
NIPC é inscritível:  NCI=NPI=w, INP=ICP=y
PIQD é inscritível:  IPQ=IDQ=w, IQP=IDP=z
MIQA é inscritível:  QAI=QMI=x, MAI=MQI=x


Peraí! NBI = x = MAI?? Isso não vale, algo está errado. Fiz essa
construção num programa de geometria dinâmica e o quadrilátero MNPQ só é
inscritível em casos MUITO particulares...

[]'s

Alexandre Tessarollo



Re: RES: ajuda em um problema

2001-10-09 Por tôpico Alexandre Tessarollo

Solução:
Faz a figura para ficar mais fácil de ver...
Como M, N, P e Q são as projeções e I  sobre  os lados AB, BC, CD, DA
temos que:
Os quadriláteros BMNI, NIPC, PIQD, MIQA são todos inscritíveis já que
possuem angulos opostos somando 180 graus. 
Como o quadrilátero ABCD é inscritível, temos que: CBD=CAD=x (ler
angulo BCD...)
ABD=ACD=y, BAD=BDC=z, ADB=BCA=w. 
Como:
BMNI é inscritível: MBI=MNI=y, NBI=NMI=x  
NIPC é inscritível:  NCI=NPI=w, INP=ICP=y
PIQD é inscritível:  IPQ=IDQ=w, IQP=IDP=z
MIQA é inscritível:  QAI=QMI=x, MAI=MQI=x


Peraí! NBI = x = MAI?? Isso não vale, algo está errado. Fiz essa
construção num programa de geometria dinâmica e o quadrilátero MNPQ só é
inscritível em casos MUITO particulares...

[]'s

Alexandre Tessarollo

PS: Agradeço a observação do Alexandre Terezan (feita em off) quanto ao
erro



Re: ajuda em um problema e Extra

2001-10-08 Por tôpico Alexandre Tessarollo



 harold wrote:
 
 seja ABCD um quadrilátero convexo inscrito num círculo e seja I  ponto
 de intersecção das suas diagonais. As projeções de sobre os lados AB,
 BC,CDe DA
 são respectivamente ,M,N,P e Q. Prove que o quadrilátero MNPQ é
 circunscrítivel a um círculo com centro em I.

Projeções de QUEM sobre os lados? Tentei projetar I e a afirmação
tornou-se falsa (leia-se achei diversos contra-exemplos). Fiz o mesmo
com O e tampouco funcionou... Seja claro e específico, plz.

[]'s

Alexandre Tessarollo

PS: Aproveitando a deixa, passo mais um:

Sabendo que sen 2A, sen 2B e sen 2C estão em PA nessa ordem, demonstrar
que tan (B+C), tan (C+A) e tan (A+B) também estão em PA nessa ordem.



Re: opf

2001-09-29 Por tôpico Alexandre Tessarollo



pichurin pichurin wrote:
 
  --- romenro [EMAIL PROTECTED] escreveu:  Eu sou
 de São José dos Campos, estou no 2º colegial,
 
  achei a prova facil, com excessão da questão 16 e
  17. Vc
  sabe resolve-las?
 
 
 
 __
  AcessoBOL, só R$ 9,90! O menor preço do mercado!
  Assine já! http://www.bol.com.br/acessobol
 
 sim, consegui resolvê-las mas acabei errandoa 16
 mas na 17 posso te ajudar:
  QV1=qV2
 ((M(V1)^2)/2)= (m(V2)^2)/2
 
 aí é só resolver o sistema assim
 (Q/q)=(V2/V1)
 ((v1)^2/(V2)^2)= M/m
 V2/V1=(M/m)^(1/2)
 
 portanto:
 
 (Q/q)=(M/m)^(1/2)
 
 
 
 

Pô... Será q vcs não podem indicar algum site com a prova? Ou pelo
menos  colocar os enunciados dessas duas (16 e 17) p/a gente discutir...

[]'s

Alexandre Tessarollo

PS: Quanto a ser off-topic, creio q discussões sobre Física são (como já
foram) sempre bem-vindas... Correto, Nicolau?



Re: Geometria

2001-08-31 Por tôpico Alexandre Tessarollo



Eduardo Casagrande Stabel wrote:
Ola pessoal!
Eu tenho duas perguntas sobre geometria.
1. Sejam A e B dois pontos distintos do plano, qual o nome, propriedade
ou
quaisquer coisa relacionada a figura formada por todos os pontos P
de tal
forma que o angulo A^PB seja constante?


 Ao conjunto L dos pontos do plano que tm
uma determinada propriedade P, tal que todo elemento de L possui a propriedade
P e apenas os elementos de L possuem a propriedade P d-se o nome
de lugar geomtrico ou, em ingls e latim, locus. Um
exemplo  a circunferncia. Todos os pontos da circunferncia
possuem a propriedade de distar R do centro O, e apenas os pontos da circunferncia
possuem esta propriedade.
 O lugar geomtrico que vc procura chama-se
Arco Capaz. Dado um ngulo XYZ e um segmento AB, constri-se
assim:
1) transporte o ngulo XYZ para o segemento AB, ou seja, AB ser
um dos lados do ngulo e A ser o vrtice. Se vc sabe
como fazer isto, pule para o passo 2.
 a) Com um distncia qualquer no compasso, faa
um arco centrado em Y e que corte YX e YZ.
 b) Com esta MESMA distncia, centre em A e
faa a circunferncia. (Poderia ser s um arco que
cortasse AB e fosse um "pouquinho" maior que o arco original, mas daqui
a pouco vc vai entender o porqu da circunferncia completa).
 c) Seja T e U as interseces do arco
(construdo no item a) com YX e YZ, respectivamente.
 d) Marque a distncia TU no compasso.
 e) Seja C a interseco da circunferncia
(construda no item b) com o segmento AB.
 f) Centrando em C, marque a distncia (TU)
na circunferncia (construda no item b). Surgiro
os pontos D ("acima" de AB) e E ("abaixo" de AB).
 g) Agora, temos BAD = BAE = XYZ
2) Tomemos o ngulo BAD = XYZ (D "acima" de AB). Construa a perpendicular
a AD que passa por A.
3) Construa a mediatriz de AB.
4) Seja O a interseco da mediatriz de AB com a perpendicular
(construda no item 2).
5) Centrando em O e com o compasso "indo" at A, faa
o arco "at" B. Note que vc pode tanto fazer um arco grande como
um arco pequeno. Faa o arco "para baixo". (Se o seu ngulo
for agudo, ser o arco maior. Se for obtuso, ser o arco
menor.)
6) Repita os passos 2 a 5, sendo que no passo 2 voc dever
pegar o ngulo BAE ao invs de BAD (E "abaixo" de AB) e no
passo 5, voc dever construir o arco "para cima".

 A juno destes dois arcos d
o Arco Capaz que enxerga AB sob um ngulo XYZ. A figura no
final parece um oito deformado se XYZ for agudo, uma quase-elipse achatada
se o ngulo for obtuso e  exatamente uma circunferncia
se o ngulo for reto.

 Prova:
 Observe que o ngulo OAB  comlementar
de BAD = XYZ por construo. Seja M o ponto mdio
de AB. O ngulo AMO  reto pois MO  mediatriz de AB,
por construo. Assim, como num tringulo a soma dos
ngulos internos  180, no tringulo AMO, o ngulo
AOM dever ser igual a BAD = XYZ.
 Como AM=MB, MO=MO e os ngulos AMO=BMO, temos,
por lado-ngulo-lado, que os tringulos AMO e BMO so
congruentes e, em particular, os ngulos BOM=AOM=XYZ. Ou seja, o
ngulo AOB  2*XYZ.
 Mas AOB  o ngulo central do arco (construdo
em 5). Logo, qualquer ponto neste arco enxergar AB sob um angulo
de AOB/2 = (2*XYZ)/2 = XYZ.
 Repete-se a prova para o "outro" arco (construdo
em 6) e est provado, CQD.

 Quaisquer dvidas ou comentrios,
estamos a.
[]'s
Alexandre Tessarollo
PS: No passo 5, na hora de construir o arco "para cima" ou "para baixo",
basta lembrar que o arco tem de estar no lado "oposto" ao lado em que o
ngulo foi construdo, com relao 
AB. Ou seja, se voc construir BAD "acima" de AB, o arco dever
ser "abaixo", e vice-versa.

2. Temos um triangulo equilatero ABC, dentro do triangulo se tracam
tres
segmentos, cada um partindo de um lado, que nao se cruzam mas estao
um
apoiados nos outros. Mostrar que se os tres segmentos tem o mesmo
comprimento, o triangulo do meio e' tambem equilatero.

Eu ia anexar uma figurinha pro 2, mas tem gente que
nao consegue abrir.

A

/\

/ \

/ \

/ \

/ \
 /
\
 /
\
 /
x Q
 /
\
R x
\
 /
\
 /
\
 /
\
/__x___\
B
P
C

 Hum, talvez mais atrapalhe do que ajude, mas vamos
ver se eu entendi: Sejam X, Y e Z as interseces de AP e
BQ, BQ e CR, CR e AP, respectivamente. Ento, se ABC  equiltero
e AX=BY=CZ, ento o tringulo XYZ tambm  equiltero.
 Bem, se for isto, acho que at j vi
essa questo antes. S que no sei se vi a soluo
e, se vi, no lembro. Vou pensar um pouco mais... :-)
Eduardo Casagrande Stabel.

Mais uma vez,
[]'s
Alexandre Tessarollo


Re: En: Undelivarable: Re:

2001-07-24 Por tôpico Alexandre Tessarollo




 Ahhh, agora entendi o que vc quis dizer... Minha
msg (abaixo) chegou numa boa na lista, mas eu recebi o tal e-mail do mandic...
Bem, neste caso, s esperando uma resposta do nosso moderador mesmo...
Nicolau, por favor manifeste-se o quanto antes...
[]'s
Alexandre Tessarollo
Alexandre Tessarollo wrote:

 Hum, estou na lista j h um bom tempo,
mas acho q vc estava bem antes... Provavelmente, no incio dos tempos
desta lista, ela era hospedada no servidor Mandic, ou, pelo menos, o e-mail
era @mandic. Experimente atualizar seu livro de endereos / address
book para [EMAIL PROTECTED]
 Creio que isto deva resolver seus problemas imediatos,
de qualquer forma, procure/espere uma resposta do Nicolau, responsvel
pela lista. Com certeza ele saber explicar muito melhor do que
eu ;-)
[]'s
Alexandre Tessarollo
Jose Paulo Carneiro wrote:
Alguem sabe
me explicar o que significa esta mensagem que ja recebi mais de uma vez?JP
- Original Message -
From: [EMAIL PROTECTED]
To: Jose
Paulo CarneiroSent: Monday, July 23, 2001 9:11 PMSubject:
Undelivarable: Re:
Caro internauta,
A mensagem enviada para [EMAIL PROTECTED]no
pode ser entregue porque o domnio @mandic.com.br
mudou para @osite.com.br.
Por favor reenviesua
mensagem conforme exemplo abaixo:
DE : obm-l>@mandic.com.br
PARA : obm-l>@osite.com.br
O SITE,
SEU LUGAR NA INTERNET


O SITE recomenda:
Acesse o Atendimento
On-Line, via Chat.
Voc no
precisa mais ter 2 linhas telefnicas para conectar
e falar com
o Suporte Tcnico.
Agora voc
pode navegar e ser atendimento ao mesmo tempo.
Clique no endereo
abaixo:
http://cadastro.osite.com.br/suaconta/chatosite.shtml>












Re: AJUDA!!! ONDE encontro?

2001-07-02 Por tôpico Alexandre Tessarollo


Com paciência, tempo e dedicação, em sebos. Afora isso, livros da MIR,
conforme discutido aqui anteriormente, só nos sites:

www.livifusp.com.br  - Livraria de Física da USP, tem alguns vários da
MIR.
urss.ru  - Editora russa que herdou MUITOS livros da MIR. Há
em inglês, espanhol, francês, russo..

Se a minha parca memória não está me traindo, alguém até citou uma
loja/livraria/sebo/sei-lá-o-que que vendia livros da MIR, mas era em
Sampa e eu sou carioca, então nem cheguei a anotar o endereço. Se vc
quiser, peça p/a lista ou dê uma checada nos arquivos da lista.

[]'s

Alexandre Tessarollo

 Marcos Eike wrote:
 
 Pessaol onde eu encontro o livro fundamentos de física elementar da
 editora MIR, se eu não me engano os autores são 6 russos
 
 Ats,
 Marcos Eike



Re: 0,8... + 0,1... = 1 ?!

2001-06-24 Por tôpico Alexandre Tessarollo


Uma maneira de vc mostrar que dois números REAIS são distintos é
mostrar que existe pelo menos um número entre eles. Caso contrário, são
o mesmo número.

Assim, por exemplo, 1 e 2 são distintos pq posso, por exemplo, ver que
1,7 está entre eles. Já 4 e 4 são o mesmo número pq não consigo achar
nehum número que esteja entre 4 e 4.

Com 0,9... e 1, é a mesma coisa que 4 e 4: não conseguimos achar
alguém entre eles., Logo, são o mesmo número.

[]'s

Alexandre Tessarollo

PS: Vc talvez esteja com uma dúvida que eu tive por muito tempo. Era uma
raciocínio em que eu fazia uma analogia dos reais com os inteiros para
matar o argumento que eu expliquei acima. (Não) Funcionava assim:

Pegue os números 1, 2, 3 e 4, por exemplo. 2 e 4 são distintos pq há o
3 entre eles. 1 e 2, APESAR de não haver ninguém (inteiro) entre eles,
são distintos* e 1 é o número que vem IMEDIATAMENTE antes de 2. 

Analogamente, pegue os números 0,999... e 1. Eles são distintos* e
0,999... é o número IMEDIATAMENTE antes de 1. A diferença 1-0,999...
seria uma infinitesimal**, o número IMEDIATAMENTE depois de zero.

*O problema que eu não percebia era como saber que os números eram
distintos.

**Esse era o nome que eu dava para tal valor, usando o bom-senso de
chamar algo muito pequeno (1-0,999...) por um nome que lembrasse algo
tão infinitamente pequeno quanto a diferença. Contudo, hoje sei que está
errado porque parte de um princípio (1 diferente de 0,999...) errado.

Ralph Costa Teixeira wrote:
 
 Oi, Gustavo.
 
 Na minha opiniao, 0,...=1. Sim, eh isso mesmo eh 1. Nao tem
 nada entre esses dois numeros. O primeiro vale 1, o segundo tambem. :)
 
 Eu mandei uma mensagem uma vez que tentava esclarecer um pouco
 porque as pessoas acham isso surpreendente (nao tanto porque eh 1 -- para
 isso o seu argumento funciona, e hah outros equivalentes como o do
 Daniel). Procure no arquivo da Olimpiada, em Abril de 2000, sob o titulo
 O Dia que nao acaba.
 
 Abraco,
 Ralph
 
 P.S.: O arquivo da Olimpiada que o Nicolau mantem com tanto carinho estah
 em http://www.mat.puc-rio.br/~nicolau/olimp/obm-l.html
 
 P.S.2: Alias, esta questao tambem jah havia aparecido antes em Maio de
 1999. Veja tambem
 
 http://www.mat.puc-rio.br/~nicolau/olimp/obm-rj.1999/threads.html#00143
 sob o titulo Dizima ou nao-dizima.
 
 On Sat, 23 Jun 2001, Augusto Morgado wrote:
 
  Eu acho estranho que alguns achem estranho 0,999...=1 e não achem
  estranho 0,333...=1/3.
  Morgado
 
  Gustavo Nunes Martins wrote:
  
   8/9 = 0,888...
   1/9 = 0,111...
   0,888... + 0,111... = 0,999...
   8/9 + 1/9 = 9/9 = 1
   Entao:
   0,999... = 1
  
   Nao entendo que 0,999... seja IGUAL a 1. Suponho que seja diferente de
   1. Alguem pode me explicar o que esta coisa significa?
  
   Atenciosamente,
   Gustavo
 



Newton (de novo)

2001-06-21 Por tôpico Alexandre Tessarollo

Bueñas!

Mandei umas perguntas recentemente, mas, como todos estavam hã,
empolgados com a OBM, elas (as perguntas) não receberam nenhuma
atenção... snif ;-(

1) Dado um ângulo de vértice A e um ponto E no interior do ângulo,
ache B pertencente a um dos lados (do ângulo) de forma que BD seja
mínimo, onde D é a intersecção da reta BE com o outro lado do ângulo.
Vide figura anexa.

2) Dado uma quadrilátero ABCD circunscrito a uma circunferência de
centro O, conforme a figura, prove que a reta que une os pontos médios
das diagonais passa pelo centro O do círculo.

[]'s

Alexandre Tessarollo

PS: Segundo qum me passou, ambas as questões teriam sido formuladas por
Newton.

 Newton.zip


Re: Questões de combinatória/jogos

2001-06-21 Por tôpico Alexandre Tessarollo



Marcelo Rufino de Oliveira wrote:

 Abaixo vão 2 problemas de combinatória/jogos que eu ainda não consegui
 fazer.
 Já mandei estas mesmas duas questões anteriormente para a lista mas
 infelizmente ninguém se manifestou... vamos ver se desta vez alguém pode me
 ajudar.
 Já agradeço, de antemão, aos participantes da lista que tentarem fazer algum
 dos problemas, pois estes não são elementares.

 1) O conjunto {1, 2, ..., 49} é particionado em 3 subconjuntos disjuntos.
 Mostre que ao menos um dos subconjuntos possui três números a, b e c tais
 que a + b = c.


Hum, vamos ver...
1a hipótese: Separamos de acordo com o resto na divisão por 3.

Assim, temos o grupo que resta 1, o que resta 2 e o que não resta nada. Neste
último, basta pegar números a=3k, b=3j e c=3(k+j). Naturalmente, k e j são
naturais não-nulos, k é diferente de j e k+j17. (Isto para que a,b e c estejam
no conjunto original {1,..,49})

Ih, tô vendo que vai dar um certo trabalho e eu tenho aula daqui a dez
minutos... Bem, veja se consegue mostrar o que o problema pede pensando nessas
possibilidades. Talvez tenha uma maneira mais direta, não sei. Vou ver se até
amanhã eu consigo resolver e digitar tudo.

[]'s

Alexandre Tessarollo

PS: Sei que não é a resolução completa, mas de repente ajuda... :-)



 2) Dado um retângulo 1993x1994, dois jogadores (um de cada vez) escreve os
 números 0 ou 1 nas casas. Quando o tabuleiro  está completo seja A o máximo
 valor das somas das 1993 linhas e B o máximo valor das somas das colunas. No
 caso em que A  B o primeiro ganha, no outro caso B ganha. Quem possui uma
 estratégia vencedora?

 Falou,
 Marcelo Rufino




Re: En: Ligar os Pontos

2001-06-21 Por tôpico Alexandre Tessarollo




Hugo Iver Vasconcelos Goncalves wrote:
A vai um desafio
que um colega meu viu e anda quebrando a cabea do pessoal lah da
minha turma, ligar usando linhas os pontos A, B, C aos pontos D, E, F sem
que essa linhas se cruzem. Isso eh mesmo possvel???A
.
B .
C .D .
E .
F .A-D uma linha, A-E uma linha, A-F uma linha,
B-D uma linha, B-E uma linha etc.Ser que dava
pra algum mandar um desenho dessa soluo (se eh
que ela existe)???

Caro Hugo,
 esse  um velho problema que no tem
soluo. Quando me foi apresentado, dizia que vc tinha trs
casas (A, B e C) e trs servios como gua, luz e telefone
(D, E e F). A idia era ligar cada casa aos trs servios
de modo que as ligaes no se cruzassem. Ou seja,
exatamente o mesmo enunciado que vc passou, s que com uma historinha.
 A prova de que tal problema no tem soluo
s se aprende na faculdade, num ramo da Matemtica chamado
Topologia. No s se prova que no plano euclidiano (o nosso
plano habitual, algo como uma face de um folha de papel) no h
soluo, como que o problema pode ser resolvido num toro
(a superfcie externa de uma cmara de ar de um pneu de caminho).
Provavelmente algum j deve ter te mostrado uma "soluo"
que apresentava todas as ligaes conforme pedidas, exceto
por uma, que, por exemplo, sai de E e chega em B por "trs" da folha.
 Contudo, ainda no comecei esta parte de topologia
- afinal, ainda estou no terceiro perodo :-) Agora, se algum
aqui na lista se dispuser a demonstrar isto...
[]'s
Alexandre Tessarollo




Re: Equacao irracional

2001-06-18 Por tôpico Alexandre Tessarollo


Hum, se o Érico quiser presentear uma matemática (ou alguma pessoa que
goste MUITO de Mat), creio que o Fábio respondeu muito bem. Já se for
para alguém que não simpatiza muito, recomendo uma que ganhei da minha
namorada:

x^2 + (-2amo)x + [(amo)^2 - (te)^2] = 0

Uma equação do segundo grau ax^2 + bx + c = 0. Utilizando o bom e velho
Báskara, temos as raízes
x1 = amo + te
x2 = amo-te

[]'s

Alexandre Tessarollo

PS: naturalmente, a, m, o, t, e são constantes reais.

Fábio Dias wrote:
 
 -Mensagem original-
 De: [EMAIL PROTECTED] [mailto:[EMAIL PROTECTED]]Em nome de
 Erico Furukawa
 Enviada em: domingo, 17 de junho de 2001 00:44
 Para: [EMAIL PROTECTED]
 Assunto: Equacao irracional
 
 Estou precisando de uma ajuda para um desafio:
 ''Elaborar uma equacao irracional cuja V={amor}, e
 cuja raiz estranha seja {odio}.''
 Se poderem me ajudar eu ficarei muito agradecido.
 
 Não está muito bem definido o que o desafio quer dizer com as duas palavras.
 Uma hipótese é que as palavras sejam números em bases suficientemente altas,
 já que depois dos do 0, 1, ..., 9, se passa a utilizar letras.
 
 Várias escolhas de base são naturais para o problema:
 - base 23 (*todos* os dígitos são representados por letras a..z sem k, w, y)
 - base 26 (*todos* os dígitos são representados por letras a..z com k, w, y)
 - base 33 (além de 0..9, os outros dígitos são representados por letras a..z
 sem k, w, y)
 - base 36 (além de 0..9, os outros dígitos são representados por letras a..z
 com k, w, y)
 
 Como são duas raízes que estão determinando a equação, é natural que tenha
 dois parâmetros. Vou tomar como equação-modelo sqrt(x + a) = b - x. O lado
 direito da inequação se torna x - b no caso odio, já que é raiz estranha.
 
 Os valores de a e b, obtidos no Maple, foram:
 BASE 23: a = 5915129246, b = 95764
 BASE 26: a = 14378381328, b = 146682
 BASE 33: a = 52509302416, b = 649234
 BASE 36: a = 102815878548, b = 864602
 
 Os valores obtidos não são nada pequenos, mas funcionam.
 
 []s,
 
 
 Fabio Dias ([EMAIL PROTECTED], ICQ# 31136103)
   RPG em Revista: A sua revista virtual de RPG!
  -- http://www.rpgemrevista.f2s.com --



Ângulo do Newton

2001-06-12 Por tôpico Alexandre Tessarollo


Uma questão que, segundo me passaram, foi formulada por Newton:

Dado um ângulo [agudo(?)] Â e um ponto E [interior ao ângulo Â]
encontre B de forma que [o comprimento] BED seja mínimo.
[adendos meus, para quem não entender a figura].

   /
 B
/  \
   / \
 / \
   /   E
 /\
   /__\_
A  D


[]'s

Alexandre Tessarollo




Quadrilátero inscrito

2001-06-12 Por tôpico Alexandre Tessarollo


Dado um retângulo ABCD e os pontos P, Q, R e S pertencentes aos
segmentos AB, BC, CD e DA, respectivamente, de forma que DP=AQ=BR=CS= x.
Determine x de forma que a área de PQRS seja mínima.

[]'s

Alexandre Tessarollo




Quadrilátero do Newton

2001-06-12 Por tôpico Alexandre Tessarollo


Aproveitando o ensejo, outra questão do Newton:

[Prove que,] Dado um quadrilátero ABCD circunscrito a uma
circunferência LAMBDA, a reta que une os pontos médios das diagonais [AC
e BD] passa pelo centro de LAMBDA.

É, dessa vez dica meio difícil fazer uma figurinha em ASCII...

[]'s

Alexandre Tessarollo




[Fwd: [EMHL] Mir Publishing]

2001-06-07 Por tôpico Alexandre Tessarollo


A resposta que obtive na outra lista..

[]'s

Alexandre Tessarollo

Paul A. BLAGA wrote:

 On Wed, 6 Jun 2001, Alexandre Tessarollo wrote:
 The Mir Publishers does not exist anymore, as far as I know, but there is
 another publishing house which continues their work, EDitorial URSS. The
 home page is, as far as I remember, http://urss.ru.

 They still have a lot of books from the former MIR.

 Paul Blaga
 
  Does anybody knows about MIR publuishing? Do they have some kind of
  home-page, some site where we can find their books or any indication?
 
  []'s
 
  A. Tessarollo
 
  PS: I know this message may be considered as an off-topic one. It's
  just MIR has wonderful books about Maths and particulary about Geometry.
  :-)
 
 
 
 
  Your use of Yahoo! Groups is subject to http://docs.yahoo.com/info/terms/
 
 

 --
 =
 Dr. Paul A. Blaga
 Babes-Bolyai University of Cluj-Napoca
 Faculty of Mathematics and Computer Science,
 Department of Pure Mathematics
 1, Kogalniceanu Street,
 3400 Cluj-Napoca, Romania
 e-mail: [EMAIL PROTECTED]



 Your use of Yahoo! Groups is subject to http://docs.yahoo.com/info/terms/




Re: O que eh isso????!!!!

2001-06-04 Por tôpico Alexandre Tessarollo


Imagine um ponto. Agora arraste este ponto e vc terá um segmento
medindo, hã, a. Arraste este segmento até obter um quadrado de lado
a. (Note que começamos com o aidmensional, fomos para o unidimensional
e depois para o bidimensional, sempre acrescentando uma dimensão.)
Arraste o quadrado e vc terá um cubo (tridimensional). E, por que não?,
arrastar o cubo e chegar num cubo quadridimensional? Como o Leonardo
Motta já disse, quando se fala em
espacos com mais de 3 dimensoes, referem-se por hiper [tanto o espaço
como os objetos nele contidos], assim, temos hiperplanos, hipercubos,
hiperetc.. :-) Esta é a melhor visualização que EU consigo para explicar
a pergunta.

Outra maneira é com geometria analítica: Se estivermos em R^1 (a reta
real), as possíveis coordenadas feitas com 0 (zeros) e 1 (uns) (em
que há pelo menos um 1) são (1) e (-1). Já em R^2(o plano), temos
(1,0), (-1,0), (0,1), (0,-1). Em R^3 (o espaço como estamos acostumados)
teremos (1,0,0), (-1,0,0), (0,1,0), (0,-1,0), (0,0,1), (0,0,-1). Por
fim, em R^4 (o hiperespaço), teremos (1,0,0,0), (-1,0,0,0), (0,1,0,0),
(0,-1,0,0), (0,0,1,0), (0,0,-1,0), (0,0,0,1), (0,0,0,-1). Note que é
necessário adicionar uma coordenada a cada acréscimo de dimensão e, por
mais estranho que pareça, eu estou o tempo todo com eixos ortogonais
dois a dois. Ou seja, assim como no espaço os eixos x, y e z são
perpendiculares uns aos outros, também no hiperespaço os eixos x, y, z,
w serão ortogonais. (Não, não é difícil imaginar. É IMPOSSÍVEL! Vc
precisaria ter um terceiro olho para ver R^4. Ou, mais genericamente, vc
precisa de n-1 olhos para ver R^n.)

[]'s

Alexandre Tessarollo

PS: De acordo com a minha última observação, em quantas dimensões as
moscas enxergam? :-P

PS2: Vale lembrar que eu trabalhei o tempo todo no espaço euclidiano,
bem comportado, etc, etc, ect. Só não sei qual o termo mais adequado
para designar R^n, com n3, se hiperespaço ou hiperplano. Tampouco sei
se o quarto eixo ortogonal é comumente chamado de w, ou sequer se recebe
um nome especial. Corrijam-me caso tenha cometido algum erro.


 Hugo Iver Vasconcelos Goncalves wrote:
 
 Alguém poderia explicar o que eh  um hipercubo tetradimensional ???



Re: Olimpíada universitária e Criptografia

2001-06-04 Por tôpico Alexandre Tessarollo


Só para ser preciso: o nome do livro que o JP indicou é Números
Inteiros e Criptografia RSA do Severino C. Coutinho, editado pela SBM
na Coleção Computação e Matemática (SCM) e vendido por R$25,00.

[]'s

Alexandre Tessarollo

PS: Esse e outro (excelentes) livros podem ser adquiridos na sede da
SBM, no Impa, ou em outros lugares. Consulte a lista de endereços e
(principalmente) a de livros na página da SBM, www.sbm.org.br

**
JP
**

Claro, existe o excelente livro do Collier, isto eh: S.C.Coutinho:
Criptografia SRA e Teoria dos Numeros (ou algo
parecido), editado pela SBM.
JP

**
[Davidson Estanislau]
**

   Caros amigos, gostaria da ajuda de vocês:

   Gostaria primeiro de saber se existe algum livro (em
portugues) sobre criptografia;
   Também gostaria que alguém traçasse um programa para a
Olimpíada Universitária.

   Obrigado.

   Davidson

**



Re: O Jogo Vida - Adendo

2001-05-30 Por tôpico Alexandre Tessarollo


Só um detalhe: no arquivo que mandei anexo, TODAS as casas possuem oito
vizinhos.  A figura está num toro. Ou, para quem não conhece toro: as casas da
extrema direita, por exemplo, também possuem como vizinhas as casas da extrema
esquerda. Da mesma forma, as casa de cima e debaixo são vizinhas. Quanto às dos
cantos, basta observar que estão ao mesmo tempo numa lateral e na parte superior
ou inferior. Espero ter sido claro.

[]'s

Alexandre Tessarollo




Re: Problema de Geometria

2001-04-17 Por tôpico Alexandre Tessarollo

Como admirador e estudioso da geometria, s tenho a elogiar sua prova. Realmente
bonita. Meus sinceros parabns!

[]'s

Alexandre Tessarollo

Edson Ricardo de Andrade Silva wrote:

 Antes de mais nada, vamos a uma breve apresentacao. Sou
 participante desta lista ha uns meses e nao tenho tido muito tempo para
 debater com vcs aqui da lista pois estou condenado a terminar de escrever
 minha tese de mestrado aqui na UFC (Na Area de Computacao Grafica,
 precisamente na area de Modelagem Digital de Terrenos) e o tempo me
 falta... no entanto, acompanho ativamente essa lista como ouvinte.
 Bem, esse problema de geometria me eh particularmente
 interessante. Lembro-me que no tempo do 2 grau, quando eu participava das
 Olimpiadas de Matematica, havia me deparado com esse problema. O ataquei
 com todas as ferramentas que eu dispunha na epoca (geom. plana,
 analitica,transformacoes geometricas...) e nao consegui resolve-lo. Talvez
 a minha decepcao maior foi nao ter encontrado uma solucao atraves de
 geometria plana simples...
 Hoje, porem, quase que num reflexo (apesar de eu estar um pouco
 enferrujado em resolver problemas de olimpiadas), encontrei uma solucao
 bem simples para o problema. Ai vai:

 - Considere um novo triangulo B'C'D' como a rotacao de 90 graus do
 triangulo BCD em torno de D e no sentido horario (considerando o pentagono
 ABCDE descrito non sentido anti-horario). Tal rotacao faz o lado C'D'
 coincidir com o lado ED, pois o angulo CDE eh reto e C'D' = CD = ED. Alem
 disso, BD = B'D' e o angulo BDB' eh reto.(***)

 - Nao eh dificil observar que o quadrilatero ABEB' eh um
 paralelogramo:
 Observe que EB'= BC = BA. Entao falta provar que EB'// BA. Chame o
 angulo BCD = B'ED = x. Chame a intersecao do prolongamento de EB' com BC
 (ou com o possivel prolongamento de BC) de G. O quadrilatero GCDE eh
 inscritivel, pois B'ED = GCD = x. Como EDC eh reto, temos que ter o angulo
 EGC tambem reto. Logo, os lados EB' e BA sao paralelos, pois fazem os
 mesmo angulos (retos) com BC.

 - Agora eh simples. As diagonais B'B e AE do paralelogramo ABEB' se
 cruzam, obviamente, em M, com BM = MB'. Acontece que, como vimos
 anteriormente, o triangulo BDB' eh retangulo em D e isosceles (***), logo
 a altura relativa ao vertice D, ou seja DM, eh igual a metade da
 hipotenusa BB', ou seja BM. E obviamente o angulo DMB eh reto, pois DM
 eh altura.
 CQD.

 Eh isso ai gente, espero poder ter ajudado.

 PS : Se algum participante da lista tiver interesse na area de Computacao
 Grafica, precisamente nas areas de Geometria Computacional, Modelagem
 Digital de Terrenos, Visualizacao 3D e quiser manter contato, sinta-se
 livre! :)

 Abracos,
 Edson Ricardo

  On Fri, 13 Apr 2001, Marcio A. A. Cohen wrote:

Como se falou um pouco de complexos aqui, segue abaixo um problema
  interessante de geometria. Interessante no sentido de ser um problema
  conhecido, que eu acho bem dificil de se resolver por geometria plana
  simples, e bem facil de se resolver com auxilio de numeros complexos (e o
  melhor, eh desses que com geometria analitica convencional continuam
  dificeis!):
 
Eh dado um pentagono convexo ABCDE. Sabe-se que AB=BC,  CD=DE, e os angulos
  internos B e D do pentagono sao de 90 graus. Seja M o ponto medio do lado
  AE. Demonstre que os segmentos DM e BM sao iguais, e que o triangulo DBM eh
  retangulo.
 
Abracos,
Marcio
 
  PS: Solucoes simples por geometria plana sao bem vindas, pra desbancar o meu
  "dificil de se resolver por...". se alguem quiser a solucao por complexos,
  eh soh lembrar que multiplica um vetor por 90 graus eh multiplicar por
 
  PS: Solucoes simples por geometria plana sao bem vindas, pra desbancar o meu
  "dificil de se resolver por...". se alguem quiser a solucao por complexos,
  eh soh lembrar que multiplica um vetor por 90 graus eh multiplicar por
  cis^(90) = i, e desenhar o pentagono no plano.. o resto sao soh poucas
  linhas de conta.
 
 




Re: não-euclidiano

2001-04-11 Por tôpico Alexandre Tessarollo



Benjamin Hinrichs wrote:

 Colegas,

 tenho uma pergunta que me tem intrigado e no tenho conseguido provar
 com certeza a minha idia. A pergunta : quanto vale a rea de um
 tringulo esfrico (sobre uma esfera, no sei se isso est implcito em
 "esfrico"...) de rea mxima para uma esfera de raio r. Tenho imaginado
 que  quase a metade da rea da esfera, ou seja, 2*Pi*r^2... mas no sei
 se no h algum maior... alis, no sei se fui muito claro na minha
 exposio de idias.


Hum, vamos ver se eu entendi: Vc imaginou aquele tringulo de 3 ngulos
retos na esfera, de rea T = 1/8 da superfcie E da esfera e foi movendo um
dos pontos da "base" ao longo do equador at chegar bem perto do outro. At
a eu concordo. Mas ser que vc no poderia pegar a "reta" que os une e,
como uma ala, puxa-la para baixo? Isto , faz-la varrer o hemisfrio sul?
Aplicando este racocionio, acho que se pegarmos trs pontos distintos mas
infinitamente prximos, as retas que os unem dois a dois possuem uma parte
"menor" e outra "maior". Assim, tomando os segmentos maiores, poderamos ter
um tringulo cuja rea '"quase" igual a da esfera, isto , tende a da esfera
e, para um "epsilon" aceitvel, T = E. Como eu no estudei geometria
esfrica ainda, posso estar falando as maiores asneiras da parquia.
Agradeo eventuais correes e/ou comentrios.


 Grande abrao,

 Benjamin Hinrichs

[]'s

Alexnadre Tessarollo




Re: geometrias triângulo com mais de 180o ?

2001-04-11 Por tôpico Alexandre Tessarollo



Rogerio Fajardo wrote:

 Por dois pontos passa uma nica reta no decorre do axioma das paralelas? Ou
  um axioma?

No decorre dos axiomas das paralelas. Pois, se tal axioma diz que "por um
ponto fora da reta, passa uma e somente uma reta paralela  primeira", como
vamos entend-lo, se no soubermos o que  "reta"?
Mas agora, pensando bem, eu posso ter confundido as coisas (escrever com
pressa no d certo :-)). At onde eu me lembre, pontos, retas, planos e algumas
outras coisas so "entes primitivos", isto, no so definidos, presume-se que
todos concordem sobre o que so.


 Alis, quais so os axiomas de Euclides?

De qualquer forma, para um estudo cuidadoso, com cada axioma, cada teorema, etc,
recomendo o livro que usei em Geometria I. Chama-se "Geometria Euclidiana Plana"
do Joo Lucas Marques Barbosa. Encontra-se  venda pela SBM, em vrios lugares,
inclusive no IMPA.

A pro


 From: "josimat" [EMAIL PROTECTED]
 Reply-To: [EMAIL PROTECTED]
 To: [EMAIL PROTECTED]
 Subject: Re: geometrias  tringulo com mais de 180o ?
 Date: Mon, 9 Apr 2001 23:52:26 -0300
 
 Essa tal demonstracao errada nao seria de Lagrange, em vez de Legendre?
 []s, Josimar

No sei se Lagrange tambm errou ou se sequer tentou, mas estou fazendo uma
cadeira aqui na UFRJ na qual estamos estudando os livros do Legendre, e foi nele
que estava o erro.


[]'s

Alexandre Tessarollo


  -Mensagem original-
  De: Alek [EMAIL PROTECTED]
  Para: [EMAIL PROTECTED] [EMAIL PROTECTED]
  Data: Tera-feira, 10 de Abril de 2001 22:12
  Assunto: Re: geometrias  tringulo com mais de 180o ?
 
 
  Desculpe, mas faz pouco tempo a professora de algebra linear provou
 que por dois pontos distintos passa somente uma unica reta(na euclidiana).
  Portanto ou essa definiao de axioma esta errada, ou isso de
 2pontos1reta nao  axioma.
  Alguem pode resolver este misterio???
 
 
 
  At 17:39 09/04/01 -0300, you wrote:
 
 
  Como alguns devem saber, Euclides foi o primeiro a formalizar
 a geometria e,
  para tanto, usou alguns axiomas (ou postulados) para provar cada
 teorema.
  (Axioma  algo que no pode ser provado e que o bom senso diz ser
 verdadeiro. Um
  exemplo de axioma seria o de que "por dois pontos distintos passa
 uma e somente
  uma reta")
 
  Vrias geometrias foram construdas ao longo dos tempos,
 excluindo um ou
  outro axioma. Em geral, devido a sua no-obviedade, o primeiro
 axioma a ser
  excludo era o das paralelas (dados uma reta r e uma ponto P no
 pertencente a
  r, existe um e somente uma reta paralela a r que passa por P).
 Assim surgiram as
  geometrias no-euclidianas, com vrias aplicaes tericas e
 algumas prticas.
  Resumidamente, so classificadas de acordo com a soma dos ngulos
 internos de um
  tringulo: maior que 180 ou menor que 180.
 
  A geometria esfrica (ou da esfera de Rienman)  aquela onde
 as retas so os
  crculos mximos, isto , de centro no centro da esfera e raio at
 um ponto
  desta. Com tais retas, pode-se construir um tringulo com trs
 (!!) ngulos
  retos. Imagine o meridiano de Greenwich, o de 90 graus e o
 equador. Se
  necessrio, pegue um globo terrestre.  fcil ver que o V
 postulado (o axioma
  das paralelas escrito por Euclides) no vale nessa geometria.
 
  A geometria do Plano de Poincar ( essa a geometria
 elptica?)  toma a
  regio do plano cartesiano onde y0 e adota como retas x = k e
 arcos de
  circunferncias centradas no eixo x e com raios quaisquer. Pode
 parecer estranho
   primeira vista (e realmente ), mas, assim, vc pode construir um
 tringulo com
  menos de 180.
 
  J a geometria Euclidiana foi reescrita por alguns, como por
 exemplo por
  Legendre. Legendre resolveu adotar outros postulados para
 construir a mesma
  geometria de Euclides. Contudo, ao tentar provar o V postulado,
 ele cometeu um
  erro de raciocnio que passou indetectado por anos. Isto , todos
 sabiam que
  haviam um erro na argumentao dele, mas no conseguiam ach-lo.
 Quem tiver
  acesso, vale a pena dar uma olhada nos livros dele.
 
  Espero ter sido de alguma ajuda e/ou esclarecimento e espero
 tambm que
  algum me corrija se tiver falado umas besteirinhas... :-)
 
  []'s
 
  Alexandre Tessarollo
 
  PS: Villard, vc deve (ou pelo menos deveria) estar vendo estas
 noes (e no o
  estudo aprofundado) em Geometria I com a Walcy. Estudo de
 geometrias
  no-euclidianas MESMO, s em Geometria II, que  eletiva. A
 propsito, bem
  vindo, calouro. Abraos do seu veterano.. hehe
 
  Rodrigo Villard Milet wrote:
 
   Sim ! Se voc tiver soma dos ngulos internos igual a 180, com
 certeza est
   presente o axioma das paralela

Re: geometrias triângulo com mais de 180o?

2001-04-09 Por tôpico Alexandre Tessarollo


Como alguns devem saber, Euclides foi o primeiro a formalizar a geometria e,
para tanto, usou alguns axiomas (ou postulados) para provar cada teorema.
(Axioma  algo que no pode ser provado e que o bom senso diz ser verdadeiro. Um
exemplo de axioma seria o de que "por dois pontos distintos passa uma e somente
uma reta")

Vrias geometrias foram construdas ao longo dos tempos, excluindo um ou
outro axioma. Em geral, devido a sua no-obviedade, o primeiro axioma a ser
excludo era o das paralelas (dados uma reta r e uma ponto P no pertencente a
r, existe um e somente uma reta paralela a r que passa por P). Assim surgiram as
geometrias no-euclidianas, com vrias aplicaes tericas e algumas prticas.
Resumidamente, so classificadas de acordo com a soma dos ngulos internos de um
tringulo: maior que 180 ou menor que 180.

A geometria esfrica (ou da esfera de Rienman)  aquela onde as retas so os
crculos mximos, isto , de centro no centro da esfera e raio at um ponto
desta. Com tais retas, pode-se construir um tringulo com trs (!!) ngulos
retos. Imagine o meridiano de Greenwich, o de 90 graus e o equador. Se
necessrio, pegue um globo terrestre.  fcil ver que o V postulado (o axioma
das paralelas escrito por Euclides) no vale nessa geometria.

A geometria do Plano de Poincar ( essa a geometria elptica?)  toma a
regio do plano cartesiano onde y0 e adota como retas x = k e arcos de
circunferncias centradas no eixo x e com raios quaisquer. Pode parecer estranho
 primeira vista (e realmente ), mas, assim, vc pode construir um tringulo com
menos de 180.

J a geometria Euclidiana foi reescrita por alguns, como por exemplo por
Legendre. Legendre resolveu adotar outros postulados para construir a mesma
geometria de Euclides. Contudo, ao tentar provar o V postulado, ele cometeu um
erro de raciocnio que passou indetectado por anos. Isto , todos sabiam que
haviam um erro na argumentao dele, mas no conseguiam ach-lo. Quem tiver
acesso, vale a pena dar uma olhada nos livros dele.

Espero ter sido de alguma ajuda e/ou esclarecimento e espero tambm que
algum me corrija se tiver falado umas besteirinhas... :-)

[]'s

Alexandre Tessarollo

PS: Villard, vc deve (ou pelo menos deveria) estar vendo estas noes (e no o
estudo aprofundado) em Geometria I com a Walcy. Estudo de geometrias
no-euclidianas MESMO, s em Geometria II, que  eletiva. A propsito, bem
vindo, calouro. Abraos do seu veterano.. hehe

Rodrigo Villard Milet wrote:

 Sim ! Se voc tiver soma dos ngulos internos igual a 180, com certeza est
 presente o axioma das paralelas !
   Villard !
 -Mensagem original-
 De: Rogerio Fajardo [EMAIL PROTECTED]
 Para: [EMAIL PROTECTED] [EMAIL PROTECTED]
 Data: Segunda-feira, 9 de Abril de 2001 11:40
 Assunto: Re: tringulo com mais de 180o?

 
 Isso significa que poderamos substituir o axioma das paralelas pelo
 axioma: "Existe um tringulo em que a soma dos ngulos  180"? Isto ,
 a existncia de um tringulo cuja soma dos ngulos  180 implica o axioma
 das paralelas e, consequentemente, que em todos os tringulos a soma dos
 ngulos  180?
 
 From: "Antonio" [EMAIL PROTECTED]
 Reply-To: [EMAIL PROTECTED]
 To: [EMAIL PROTECTED]
 Subject: Re: tringulo com mais de 180o?
 Date: Sun, 8 Apr 2001 18:46:03 -0300
 
  At onde eu saiba, em geometrias no euclidianas, a soma dos ngulos
 do
 tringulo pode ser tanto menor qto maior do que 180 graus.
  Mas como esta no  minha especialidade, deixo para os mestres da
 lista
 comentarem mais o assunto!
 
 - Original Message -
 From: "Rodrigo Villard Milet" [EMAIL PROTECTED]
 To: [EMAIL PROTECTED]
 Sent: Sunday, April 08, 2001 1:14 AM
 Subject: Re: tringulo com mais de 180o?
 
 
   A soma dos ngulos internos de um tringulo s  180 graus na geometria
   euclidiana. Explicanco melhor : Se voc verificar que a soma dos
 ngulos
   internos de um tringulo  180, voc s pode estar trabalhando com a
   geometria euclidiana. De fato, num tringuo esfrico, a soma dos
 ngulos
   internos do tringulo   180 graus. Mas esse tringulo no  definido
 na
   geometria plana euclidiana. Note que a prova de que a soma dos angulos
 
 180
   decorre do axioma das paralelas, que s  definido na geo euclidiana.
Certamente, se voc considerar uma geometria na superfcie de uma
 esfera,
   onde as retas so os grandes crculos, note que PAB ser um tringulo
 sim.
   Mas como nessa geometria no vale o axioma das paralelas, no podemos
   afirmar nada sobre a soma dos ngulos (s q ela180).
Abraos,
Villard!
   -Mensagem original-
   De: vinicius [EMAIL PROTECTED]
   Para: [EMAIL PROTECTED] [EMAIL PROTECTED]
   Data: Domingo, 8 de Abril de 2001 00:55
   Assunto: tringulo com mais de 180o?
  
  
   considerem a forma esfrica da Terra. tracemos duas linhas de seu
 extremo
   superior ou inferior (plo norte ou plo sul) - ponto P - at dois
 pontos
   distintos pertencentes  linha do Equador - pontos A e B. PAB pode ser
   considerado um tringulo? se 

Comprimento da Espiral Equiangular

2001-04-09 Por tôpico Alexandre Tessarollo

Bueas!

Resolvi parametrizar a espiral equiangular sozinho, no brao. Achei
que suas equaes seriam

x = Rcos^n(t)cos(nt)
y = Rcos^n(t)sen(nt)

onde R  o raio da circunferncia original, t  o ngulo formado
entre OP[n] e OP[n+1].

Todavia, quando fui calcular o comprimento com N de zero a infinito,
cheguei na srie
somatrio(cos^n(t)/n), n= 1, 2.. infinito.
pretendia achar algo em funo de t e depois fazer o limite quando t
-zero, s que ainda no consegui calcular o valor da srie. Vale
lembrar que 0tpi/2.

Para quem no sabe o que  a espiral equiangular e/ou no entendeu
minhas convenes, fiz a construo assim:

Seja C um crculo de centro O e raio R;
Construa um ngulo inscrito QOP[0] = t;
Construa vrios angulos seguidos, no sentido trigonomtrico, todos
iguais a t;
Marque P[1] como o p da perpendicular baixada de P[0] em relao a OQ;
Repita o processo sendo P[n+1] o p da perpendicular baixada de P[n] em
relao a OP[n+1].

Para facilitar a visualizao, anexo uma figura em formato wmf, como fig
de texto do word num arquivo rtf e, para quem tem no formato do Cabri e
no do SketchPad. Ainda para esses, vale a pena pedir o Locus de cada
P[n] quando P[0] ou Q se movem. Note que cada programa faz uma figura
diferente %=O

[]'s

Alexandre Tessarollo

OBS:
P[n] leia-se "P ndice n";
cos^n(t) leia-se "n-sima potncia de cosseno de t"
sen(nt) leia-se "seno do produto n vezes t"

 EspEquiang.zip


Re: Curvas de Perseguição

2000-09-28 Por tôpico Alexandre Tessarollo



[EMAIL PROTECTED] wrote:
 
 Onde está a questão ???
 
 A questão está enunciada em
http://www.dmm.im.ufrj.br/projeto/forum/calc.html

[]'s

Alexandre Tessarollo



Curvas de Perseguição

2000-09-27 Por tôpico Alexandre Tessarollo


Olá a todos!

Trago uma questão que vem me assolando já há algum tempo. A questão
está enunciada em http://www.dmm.im.ufrj.br/projeto/forum/calc.html,
com direito até a um *.gif animado p/melhor visualização. Minhas
perguntas são:
- Como se resolve tal questão? 
- É possível generalizar a questão para um polígono regular de n lados?
- Importa se a variação é discreta ou contínua? Pq?

Já fui até perguntar p/a minha professora de Cálculo II e ela disse que
é um problema clássico, resolvido apenas para alguns valores de n. Só
que ela não sabia nada além disso. Alguém pode me ajudar?

[]'s

Alexandre Tessarollo



Re: Prob de triangs antiga...

2000-09-01 Por tôpico Alexandre Tessarollo

O enunciado da questão não sei EXATAMENTE qual era, mas a resolução
abaixo refere-se à "Qual a probabilidade de, escolhidos três pontos
quaisquer do plano (distintos e não-colineares), o triângulo formado por
esses pontos ser acutângulo?" A solução me pareceu clara para o problema
em questão. Qual parte da resolução não está clara?

[]'s

Alexandre Tessarollo

PS: Insisti na resposta do N pq achei que a solução de 1/4 fosse dele,
mas quero ouvir a opinião de outros membros da lista... :-)

"Nicolau C. Saldanha" wrote:
 
 On Fri, 1 Sep 2000, Alexandre Tessarollo wrote:
 
 
 
Há um bom tempo discutiu-se aqui na lista qual seria a probabilidade
  de, escolhidos os pontos A, B e C distintos não-colineares, o triângulo
  ABC ser acutângulo. De acordo com uma resposta (do Nicolau?), seria 1/4.
  Porém, levei a questào a um grupo de amigos na faculdade e eles
  discordaram, pq a solução apresentada seria para o caso específico de um
  triângulo inscrito numa circunferência de raio unitário. Depois de muita
  discussão, a conclusão foi a seguinte:
 
"Pegue o maior lado, digamos AB, e trace os arcos de circunferência
  centrados primeiro em A depois em B, de raio AB. A figura fica
  semelhante a um olho. O ponto C NECESSARIAMENTE está dentro deste
  "olho", pois AB é, por hipótese, o maior lado. Trace agora a
  circunferência que tem AB como diâmetro. Se C estiver na circunferência,
  ABC é retângulo. Se estiver "dentro" da circunferência, será acutângulo.
  Se estiver fora da circunferência, será obtusângulo. (Para efeito de
  cálculo, as áreas de um lado e de outro do segmento AB são iguais.)
  Fazendo as contas, ou seja, fazendo área do (semi)círculo sobre a
  (semi)área "externa" do olho (como se fosse a parte branca do olho),
  chega-se a um número MUITO estranho, porém aceitável e válido para
  qualquer caso."
 
O raciocínio está certo? A solução mais antiga também está certa? Se
  for o caso, qual o erro de qual solução?
 
  Aguardando apreciação dos demais (em especial N.),
 
  []'s
 
  Alexandre Tessarollo
 
 
 Um outro membro da lista enviou este problema para a lista com duas respostas,
 uma das quais era 1/4. Minha resposta foi que o problema conforme formulado
 não faz muito sentido. Uma versão que *faz* sentido é se supusermos os três
 pontos em uma circunferência. Existem outras versões que também fazem sentido,
 como por exemplo tomar os três pontos dentro de um disco unitário.
 Estas outras versões devem ter respostas diferentes de 1/4.
 A conclusão dos seus colegas eu não entendi. []s, N.



Re: Pergunta solta

2000-07-30 Por tôpico Alexandre Tessarollo



Respondendo ao Marcelo: "Maple" é um programa de matemática que faz
MUITA coisa, entre gráficos (todos os tipos), integrais, derivadas, etc.

O único lugar que eu conheço e também onde comprei o meu, foi na UFRJ,
no Instituto de Matemática, no CT, por R$20. Só não sei se a venda é só
para alunos e professores da UFRJ ou se é aberta. Se for, não sei se é o
mesmo preço. O único lugar além do IM que eu acho que possa ter é no
IMPA.

Alexandre Tessarollo



Re: Questão das Olimpíadas

2000-07-28 Por tôpico Alexandre Tessarollo

Seja bem-vindo!

Bem, mãos à obra: 7/10p/q11/15   
Ou, reescrevendo, temos:

21/30p/q22/30   

Note que eu s;o coloquei um mesmo denominador p/ as duas frações
conhecidas, a desigualdade continua a mesma. 

Agora, se q=30, 21p22, o que não é possível. O primeiro q inteiro que
nos dará um p também inteiro é q=60. Assim, teríamos 42p44, logo p=43.

Resposta: letra E

Alexandre Tessarollo

 José Alvino wrote:
 
 Olá pessoal!
 Sou novato aqui na lista e gostaria que alguém me ajudasse numa
 questão ou me informasse onde posso encontrar sua resolução. É a
 questão 10 da 1a fase junior de 97:
 
 
 Se p e q são inteiros positivos tais que 7/10  p/q  11/15 , o menor
 valor que q pode ter é:
 
 A) 6 B) 7 C) 25 D) 30 E) 60
 
  Agradeço antecipadamente.
 




Re: Combinatoria

2000-07-20 Por tôpico Alexandre Tessarollo

Essa é uma questão de permutação circular. Fiz de duas maneiras.

Primeira maneira:

Vamos primeiro permutar todas as bolas como se estivessem uma ao lado
da outra numa prateleira. Para quem já estudou permutação c/repetição, é
fácil ver que existem N=54!/(6!8!16!24!) arrumações possíveis. Agora
"fechemos" o círculo, isto é, juntemos uma ponta da prateleira à
outra(como se a prateleira fosse maleável). Ao fazermos isto, vemos q a
arrumação q põe todas as bolas juntas de acordo com a cor, isto é,
BBAzAz..AzVV...VAmAm...Am é equivalente à
BAzAz..AzVV...VAmAm...AmB que é equivalente a várias outras.
Precisamente 54 arrumações equivalentes. Basta ver que o "ponto de
corte" da arrumação acima poderia ter sido em qualquer um dos 53 espaços
entre as bolas bem como aonde admitimos ter sido, no fim da nossa
prateleira.)
Logo, o verdadeiro número de arrumações é N/54.

Segunda maneira:

Escolha uma bola qualquer, digamos branca. Coloque-a em qualquer
posição, pois todas são equivalentes inicialemente. Agora, para colocar
a segunda bola branca, temos o lugar simétrico ao da primeira e mais
52/2=26 lugares (na verdade seriam 52 lugares, só que são simétricos
dois a dois. Logo...). Ou seja, já temos 27 possibilidades. Já podemos
perceber também que dessa maneira teremos vários casos e não chegaremos
ao resultado tão cedo.
Assim, na hora de colocar as bolas seguintes, nós "abrimos" o círculo.
Isto é, assumimos que a 1a bola colocada representa a 1a posição.
Resolvendo essa permutação normal, temos M=53!/(5!8!16!24!). Vale
lembrar que a nossa primeira bola branca NÃO é diferente das outras, ou
seja, existem 6 bolas brancas q podem ser esta primeira. Portanto, o
verdadeiro número de arrumações é M/6.

E, como podemos ver, M/6=N/54. Ou seja, ambos os raciocínios chegam a
mesma resposta e ambos estão, a meu ver, corretos.

Aguardo apreciação de todos.
Um abraço,
Alexandre Tessarollo

Ecass Dodebel wrote:
 
 "De quantas maneiras distintas podemos dispor ao longo de um
 circulo, suposto fixo, 6 bolas brancas, 8 bolas azuis, 16
 bolas verdes, 24 bolas amarelas?"
 
 O círculo fica fixo em nossa frente, mas as bolas ficam livres para serem
 rotacionadas como em uma catraca de bicicleta (acho que vocês entendem).
 
 Obrigado!
 
 Eduardo Casagrande Stabel.
 
 Get Your Private, Free E-mail from MSN Hotmail at http://www.hotmail.com



Re: circunferência x círculo

2000-07-12 Por tôpico Alexandre Tessarollo

Caro Filho,

No segundo grau aprendemos que circuferência é a linha (que une os
pontos equidistantes etc) e círculo seria a linha + a região interna.
Porém, na Faculdade eu levantei essa questão com minha professora de
Geometria I e ela me disse que para geômetras diferenciais (como ela)
circuferência e círculo seriam apenas a linha, e disco seria a linha + a
região interna. Portanto, *eu* *acho* que devemos variar os termos de
acordo com a, hã, "platéia". Todavia uma uniformização dos termos
facilitaria e muito a vida de todos, alunos, professores e pessoas
envolvidas com geometria em geral.


 Filho wrote:
 
 É muito comum se ver em livros e provas de vestibulares, perguntas do
 tipo: Determine a área da circunferência.ERRADO OU OK ? Não seria
 melhor: Determinar a área da região limitada pela circunferência ou do
 disco ou mesmo do círculo. Ou será correto se referir a uma
 circunferência como sendo um círculo? A circunferência não é um
 subconjunto do círculo? O círculo não pode ser visto como a reunião de
 várias circunferências concêntricas? Devemos fazer vista grossa?
 Gostaria de ter a opinião de vocês.



Re: Poligono e Poligonal

2000-07-12 Por tôpico Alexandre Tessarollo

Assim como circuferência é a linha (LG dos pontos etc), poligonal também
é a linha (união dos segmentos...). Da mesma forma, polígono é a região
(intersecção finita de todos os semi-planos determinados pelas retas que
contêm os segmento da poligonal). Certo?

Bruno Woltzenlogel Paleo wrote:
 
  O mesmo vale para polígono e poligonal.
 
 Kual a definição de polígono e poligonal?



Re: Problema(novamente)

2000-04-09 Por tôpico Alexandre Tessarollo



Eduardo Casagrande Stabel wrote:
 
 +/v8APAAh-DOCTYPE HTML PUBLIC +ACI--//W3C//DTD HTML 4.0
 Transitional//EN+ACIAPg- +ADw-HTML+AD4APA-HEAD+AD4- +ADw-META
 content+AD0AIg-text/html+ADs- charset+AD0-utf-7+ACI-
 http-equiv+AD0-Content-Type+AD4- +ADw-META content+AD0AIg-MSHTML
 5.00.2314.1000+ACI- name+AD0-GENERATOR+AD4-
 +ADw-STYLE+AD4APA-/STYLE+AD4- +ADw-/HEAD+AD4- +ADw-BODY
 bgColor+AD0AIw-ff+AD4- +ADw-DIV+AD4APA-FONT size+AD0-2+AD4-Estava
 pensando na funcao:+ADw-BR+AD4APA-BR+AD4AJg-nbsp+ADs-x(a , n) +AD0-
 1+AF4-a +- 2+AF4-a +- ... +- n+AF4-a+ADw-BR+AD4APA-BR+AD4-E tentava
 descobrir qual o resto da divisao de x(a , n) por n de
 forma+ADw-BR+AD4-explicita em termos do a e do n, tem jeito facil para
 fazer isso?+ADw-BR+AD4APA-BR+AD4APA-BR+AD4APA-BR+AD4-PS.
 escolhendo-se+ACY-nbsp+ADsAJg-nbsp+ADs- y(a , n , k) +AD0- k+AF4-a +-
 (k+-1)+AF4-a +- ... +- (k+- n -1)+AF4-a+ACY-nbsp+ADs- ,
 e+ADw-BR+AD4-claramente temos:+ADw-BR+AD4APA-BR+AD4AJg-nbsp+ADs-y(a ,
 n , 1)+AD0-x(a , n)+ADw-BR+AD4APA-BR+AD4-Eh facil de ver que temos y(a
 , n , k) +AD0- x(a , n) (mod n) para qualquer
 k.+ADw-BR+AD4APA-BR+AD4-PS2. a ideia eh tentar resolver um dos
 problemas que ja veio para a lista+ADw-BR+AD4-(com o a+AD0-2), ou algo
 
assim.+ADw-BR+AD4APA-BR+AD4APA-BR+AD4APA-/FONT+AD4APA-/DIV+AD4APA-/BODY+AD4APA-/HTML+AD4-


Ok, já foi traduzido do grego para o chinês. Agora será que da para
traduzir para uma língua que ao menos utilize o nosso alfabeto? :-)

Alexandre Tessarollo

PS: Só para visualizar uma figura que eu particularmente achei
interessante:

"Uma amostra de material radioativo foi prensada na forma de um cubo de
aresta a. Deseja-se envolver o cubo com material isolante de forma que
cada ponto da superfície do cubo esteja a uma distância d. Qual o volume
total de material isolante utilizado?" 

Note que a figura fica mais, hum, interessante p/ a=d.